SJT Practice Paper 1 (Official) Flashcards

1
Q
  1. Your consultant Dr Jackson has asked you to prescribe a second antibiotic for a patient who has a chest infection which has been slow to respond to initial treatment. Later that day, a pharmacist informs you that the new antibiotic is not in the hospital formulary. She tells you that the new antibiotic should not be used because of the risk of clostridium difficile infection.

Rank in order the appropriateness of the following actions in response to this situation (1= Most appropriate; 5= Least appropriate).
A. Prescribe what the pharmacist advises
B. Explain that Dr Jackson requested the antibiotic and he would be the best person to speak to about the prescription
C. Agree to contact Dr Jackson to discuss the prescription
D. Ask your specialty trainee* to review the patient to enable an informed decision
E. Do not change the prescription and make a record in the notes of the pharmacist’s concerns

A

Answer: CBDAE
Rationale: This question assesses your professionalism and how to manage your working relationships. The most appropriate action to take would be to contact your consultant (C). As Dr Jackson, your consultant, made the decision about the antibiotics, it is courteous and also in the patient’s best interest to inform him of the pharmacist’s advice. It is preferable for you to contact your consultant yourself (C) rather than expect the pharmacist to have to repeat the same information to multiple members of the same team (B). The next most appropriate action would be to ask a registrar/specialty trainee, who has more experience, to review the patient (D) as there may be complexities surrounding the patient of which you and the pharmacist are not aware. A review by a senior doctor may enable a more robust clinical decision. Option A suggests that you do what the pharmacist recommends without further discussion with her, your consultant or any other member of your team. It is Dr Jackson’s team (of which you are part) who are directly responsible for the patient’s care and there needs to be clear communication about changes to the management plan (A). Option E is not addressing the problem (E).

How well did you know this?
1
Not at all
2
3
4
5
Perfectly
2
Q
  1. On the morning ward round, your specialty trainee* said that Mrs Anderson is medically fit following her total knee replacement and could be discharged if Occupational Therapy* feel it is appropriate. The occupational therapist has assessed Mrs Anderson and believes it is safe for her to go home with a care package that has been arranged. It is now 4pm and the nurse informs you that Mrs Anderson is demanding to see a doctor as she does not feel that she is ready to go home yet. An elective admission is waiting in the day room for Mrs Anderson’s bed.

Rank in order the appropriateness of the following actions in response to this situation (1= Most appropriate; 5= Least appropriate).
A. Ask Mrs Anderson about her concerns
B. Ask a senior colleague to speak with Mrs Anderson
C. Ask the bed manager if he can find another bed for the elective patient
D. Explain to Mrs Anderson that the bed has already been allocated and she has to go home
E. Ask the occupational therapist to come and speak to Mrs Anderson with you

A

Answer: AEBCD
Rationale: This question looks at your ability to cope with pressure but also maintain a patient focus Your primary duty is to alleviate the concerns that Mrs Anderson has in relation to her discharge and reassure her that it is safe for her to go home (A). Understanding her concerns will be the first step to reassuring her that it is indeed safe for her to go home. The occupational therapist is likely to have a greater experience in dealing with questions raised by patients who have anxieties over the safety of discharge once it has been clarified that the patient is medically fit for discharge (E), although your senior staff may be able to assist you in dealing with Mrs Anderson’s concerns (B). Keeping her in hospital without addressing her issues may actually increase the risk to Mrs Anderson of suffering a hospital acquired infection and is an inefficient use of hospital resources (C). Whilst not your highest priority, you should always be aware of how patients are being allocated to beds and if it is clear there could be a delay in discharging Mrs Anderson, it would be appropriate to ask the bed manager to find a different bed for the elective patient. Trying to coerce Mrs Anderson to go home by suggesting that she is depriving another patient of a hospital bed is inappropriate as Mrs Anderson’s concerns remain central to the management of this situation (D).

How well did you know this?
1
Not at all
2
3
4
5
Perfectly
3
Q
  1. You are working on a busy paediatric ward. Your shift was meant to finish at 7pm, but it is now 9pm on a Friday, and you are still trying to complete some of your routine tasks from the day. This has happened on a number of occasions in the last month and you feel exhausted as a result. Your workload is also having a negative impact on your social life.
    Rank in order the importance of the following considerations in the management of this situation (1= Most important; 5= Least important).
    A. The impact on your own wellbeing if you are not able to take time to rest
    B. The risk to patient safety if working whilst tired
    C. Your right to finish at the designated time
    D. That your consultant may give you a poor reference if you are not completing your tasks
    E. That you are repeatedly disappointing your friends by not attending social events with them
A

Answer: BACDE
Rationale: This scenario is about maintaining a good work life balance to work effectively and provide good patient care. As a doctor, the care of the patient is your main concern (B). Your own health has to be looked after in order to provide good patient care (A). Finishing on time is suggestive of an appropriate and achievable workload and indicates good work life balance (C). Getting a poor reference may be important personally but it should have no direct or immediate impact on patient care (D). Disappointing your friends will be the least important consideration in these options as it does not affect the level of patient care and should not influence your decisions on the ward (E).

How well did you know this?
1
Not at all
2
3
4
5
Perfectly
4
Q
  1. You are working on the Surgical ward and you are about to attend theatre to observe your consultant undertake a complicated procedure. This will be a good learning opportunity for you and you anticipate being in theatre for about two hours. As you are about to leave the ward, one of the nurses tells you that a patient needs to have her medication reviewed prior to receiving her next dose in three hours’ time. He tells you that he believes one of the other FY1 doctors has been making prescription errors. You also notice one of the patients on the ward beckon you over to his bed urgently. You know from experience that the patient often just wants to have someone to talk to as he gets lonely.
    Rank the order in which the following tasks should be undertaken (1= Do first; 5= Do last).
    A. Review the patient’s dose, as requested by the nurse
    B. Respond to the patient’s immediate question or query
    C. Attend the theatre to observe the procedure
    D. Take steps to investigate the nurse’s allegations about prescription errors further
    E. Spend more time with the patient if he wants someone to talk to
A

Answer: ABCDE
Rationale: Preventing a potential medication error and ensuing patient harm should be the absolute first priority (A). The patient beckoning you may have a very valid and important question and therefore this should be acknowledged as a priority (B). Attending theatre is a good learning opportunity which will help you deliver better patient care (C). Ensuring that the nurse’s concerns about errors are addressed is very important, but not immediate (D). Spending time with a patient is desirable but can be done by others if necessary – you will need to draw it to the attention of another member of staff (E).

How well did you know this?
1
Not at all
2
3
4
5
Perfectly
5
Q
  1. It is 6pm and you are clerking a patient who is to undergo an elective splenectomy the next morning. Before he left, your consultant asked you to prescribe the antibiotics and immunisations that need to be given that evening so that surgery can proceed tomorrow. You now cannot find the folder containing the pre-operative protocols and it is not available on the intranet. Your consultant has already gone home.
    Rank in order the appropriateness of the following actions in response to this situation (1= Most appropriate; 5= Least appropriate).
    A. Seek advice from the on-call microbiologist
    B. Look in the British National Formulary* and prescribe what is suggested
    C. Refer to national guidance for pre-operative protocols
    D. Ask the nurse in charge of the ward what is normally given
    E. Seek advice from the surgical specialty trainee*

*Difficult

A

Answer: EABDC
Rationale: This question is assessing your ability to safely and responsibly clarify important clinical information and select how and where to get help. In this scenario, the antibiotics and immunisations clearly need to be administered that evening. The most logical person to contact would be a senior member of your own team who is also responsible for the patient and who would be likely to have knowledge and experience of prescribing these antibiotics in that hospital and for that consultant (E). The second most appropriate thing to do would be to contact the on-call microbiologist (A). Whilst they will certainly know which antibiotics are required and also be aware of local hospital policy, they should not be the first point of contact for routine non-emergency queries. The third most appropriate option is to look in the BNF and prescribe what is suggested (B).This is less preferable than Option A as the BNF does not take into account local policy and preferences. This behaviour would however be considered safe and appropriate (A). The next correct option is to consult with the nurse in charge on the ward (D). Whilst the nurse in charge may well be very knowledgeable and experienced, it is not appropriate to prescribe any medication purely on the advice of a nurse without consulting further with a senior medical colleague or confirming the dose etc in the BNF. National guidelines would be the last place to get information in this situation (C). Local guidelines are based on the national ones but they will include information specific to local circumstances. It is always best to use what has been locally agreed first but as a last resort, the national guidelines may be helpful.

How well did you know this?
1
Not at all
2
3
4
5
Perfectly
6
Q
  1. You are looking after Mr Kucera who has previously been treated for prostate carcinoma. Preliminary investigations are strongly suggestive of a recurrence. As you finish taking blood from a neighbouring patient, Mr Kucera leans across and says “tell me honestly, is my cancer back?”
    Rank in order the appropriateness of the following actions in response to this situation (1= Most appropriate; 5= Least appropriate).
    A. Explain to Mr Kucera that it is likely that his cancer has come back
    B. Reassure Mr Kucera that he will be fine
    C. Explain to Mr Kucera that you do not have all the test results, but you will speak to him as soon as you do
    D. Inform Mr Kucera that you will chase up the results of his tests and ask one of your senior colleagues to discuss them with him
    E. Invite Mr Kucera to join you and a senior nurse in a quiet room, get a colleague to hold your ‘bleep’*, then explore his fears
A

Answer: DCEAB
Rationale: This question places you in a challenging situation and explores your communication skills. It is not an FY1’s responsibility to break bad news to a patient in this context with incomplete information as the full results are not available yet. It would be most appropriate for a senior colleague to speak to Mr Kucera with regards to his diagnosis (D). Informing Mr Kucera that you will speak to him as soon as you get the test results back would still be appropriate as you are giving him some information, although this may not necessarily mean that you would be providing him with the diagnosis (C). It may be appropriate to discuss Mr Kucera’s fears with him, but by doing this you may not be attending to other ill patients and are asking a colleague to take on your responsibility by holding your bleep (E). It may also become a difficult conversation when you do not have full details of the results. It would not necessarily be appropriate to tell Mr Kucera that his cancer is back as this has not been confirmed (A), however it would be inappropriate to provide false hope to a patient when preliminary investigations are strongly suggestive of a recurrence (B).

How well did you know this?
1
Not at all
2
3
4
5
Perfectly
7
Q
  1. At your morning handover/briefing you are reminded by Infection Control* that all hospital staff should wear shirts with short sleeves. When wearing long sleeves, they must be rolled up and secured, particularly when having clinical interaction with patients. During your shift, you notice that your FY1 colleague always has her long sleeves down.
    Rank in order the appropriateness of the following actions in response to this situation (1= Most appropriate; 5= Least appropriate).
    A. Tell Infection Control that your colleague is not complying with their policy
    B. Speak directly to your FY1 colleague about your observation
    C. Raise your observation with the nurse in charge of the ward
    D. Do not say anything immediately but monitor the situation over the course of the next few days
    E. Discuss the situation with your specialty trainee*

*Difficult

A

Answer: BCEDA
Rationale: This question is looking at your communication with team members and patient focus. All doctors have a duty to raise concerns where they believe that patient safety is being compromised by the practice of colleagues. However, doctors strive to provide the best care possible to their patients and this situation may have arisen out of some misunderstanding. It is best therefore to speak directly to your colleague to explore the issue (B). Infection control is not just the responsibility of doctors but the whole team of staff and indeed the organisation. The nurse in charge of the ward although not a direct line manager will have a key role in ensuring standards are met and so would be a sensible person to alert (C). Your specialty trainee may be able to help address this situation, though this option is less likely to explain directly the reason for your colleague keeping her sleeves down (E). Monitoring the situation (D) is less appropriate as it does not immediately address the problem. However, it is more appropriate than involving Infection Control at this stage (A) as this would risk damaging your professional relationship with your colleague and does not explore the cause of the problem.

How well did you know this?
1
Not at all
2
3
4
5
Perfectly
8
Q
  1. You recently discharged two patients from your ward with similar names, who had undergone similar procedures. Arrangements have been made for both patients to receive follow up care in the community. When checking the patient records, you realise that you mixed up their discharge letters and sent each letter to the wrong patient. This means that each patient will receive the other patient’s treatment advice.
    Rank in order the appropriateness of the following actions in response to this situation (1= Most appropriate; 5= Least appropriate).
    A. Inform the consultant of the mix up
    B. Seek advice from an FY1 colleague about what you should do
    C. Adjust the original letters in the patients’ records
    D. Trust that health care professionals providing follow up care in the community will correct the error, apologising to the patients for the mistake
    E. Contact both patients to explain that there was a mix up
A

Answer: EABCD
Rationale: Although the patients have had similar procedures, we can be sure that they are very different individuals. Their subsequent need for information and management may be very different. The mix up with the letters causes two significant problems – the potential for the occurrence of inappropriate clinical management based upon inappropriate information, and a breach of the professional responsibilities regarding patient confidentiality. Management of the former requires some urgency. The latter requires an apology to both patients. Only answers A and E address both of these problems. Choosing Option (E) makes inappropriate clinical intervention less likely to happen. Then the consultant must also be informed of the mix up (A). The other three options fail to address the primary concerns. Seeking advice from a peer is an appropriate thing to do (B), but a more senior team member would be more appropriate. It would be appropriate to enter information that there had been an error into the case notes and indicate that the specified information did not relate to this patient (C). However, alteration of the case notes with the intention or effect of covering up an error, e.g. removing evidence, would be an extremely inappropriate and unprofessional act. Option (D) does not address the situation or put safe patient care foremost and is an inappropriate act.

How well did you know this?
1
Not at all
2
3
4
5
Perfectly
9
Q
  1. You are working on a surgical ward and are on your way to check the discharge of a post-operative patient, Joan, who is due to be transferred to a rehabilitation hospital. You have been advised by the ward manager that Joan’s bed is needed urgently for a newly arrived patient. When you arrive at Joan’s cubicle, her daughter, Allie, tells you that her mother has been complaining about her chest and is struggling with a cough. You review the observation chart and listen to Joan’s chest, which does not indicate a problem. Allie insists that her mother has a chest infection and should not be discharged.
    Rank in order the appropriateness of the following actions in response to this situation (1= Most appropriate; 5= Least appropriate).
    A. Ask the ward nurse to inform the rehabilitation hospital that Joan’s condition needs assessing on arrival
    B. Inform Allie that she should insist on a further review of Joan’s condition when she arrives at the rehabilitation hospital
    C. Advise Allie that you will delay the transfer in order to consult with a senior member of your team
    D. Advise Allie of the urgent need to discharge her mother to create space on the ward
    E. Contact the rehabilitation hospital and write detailed notes outlining Joan’s symptoms and possible investigations to send with her
A

Answer: CEABD
Rationale: This scenario is about understanding roles and responsibility, communication skills and understanding of the wider teams in the NHS. As an FY1 the roles and responsibilities include decision making with assistance. The FY1 should seek an opinion of a senior colleague, if not a consultant, when there is uncertainty whether the patient’s situation has changed and the family is involved. The FY1 should inform the patient/daughter of a possible delay while the team decides on a plan (C). Ensuring that there is good handover and communication to enable the rehabilitation hospital to conduct possible investigations is also appropriate but it defers the responsibility to the rehabilitation hospital (E). Involving the nursing team at both the discharging and admitting hospital is next in being appropriate (A). Counselling the daughter that the hospital will have been informed already and that they could expect an early review of the patient is a fair point (B). Option (D) is the least appropriate as it does not address the patient’s safety and is never an acceptable explanation if an adverse event were to occur.

How well did you know this?
1
Not at all
2
3
4
5
Perfectly
10
Q
  1. You joined a new team three months ago, and you work with two specialty trainees*, Anne and Emma, and an FY2 colleague, Malakai. You notice that the team works well when Anne is present, but when Anne is on leave or absent from the workplace, Emma and Malakai become very dominant and often undermine your decisions in front of patients.
    Rank in order the appropriateness of the following actions in response to this situation (1= Most appropriate; 5= Least appropriate).
    A. Seek advice from a more senior colleague on how to improve team relations
    B. Discuss your concerns with all team members
    C. Request to be assigned to a new team
    D. Document Emma and Malakai’s behaviour towards you
    E. Wait another week to see if the situation improves
A

Answer: BADEC
Rationale: This question is about effective teamwork and how you should communicate in a professional manner with other team members if and when you feel that your contributions, decisions and confidence are being undermined. The most appropriate option is to discuss your feelings with the whole team; some team members may need to be made aware of the effects of their behaviour (B). Seeking advice from a more senior colleague is useful (A), and will give you a sense of support and an opportunity to discuss the most appropriate actions to take, but it does not address the issue with the team members involved. Option A is however preferable to Option D, which can provide useful evidence and self-reflection of your reactions to the behaviour of your team members, but once again will not address the issue directly with the team members involved (D). It is unlikely that waiting another week will improve the situation without your direct action (E), but option E is preferable to option C, because requesting to be assigned to a new team does not address the professional issues of working effectively in a team at all, and is therefore least acceptable (C).

How well did you know this?
1
Not at all
2
3
4
5
Perfectly
11
Q
  1. You work on the Breast Surgery unit. Because of recent advances in surgical techniques, inpatient stay has dropped from five days to an overnight stay. This means that you seem to spend all your time clerking in patients and the number of learning opportunities has reduced as a result.
    Rank in order the appropriateness of the following actions in response to this situation (1= Most appropriate; 5= Least appropriate).
    A. Take on a position of responsibility as part of the junior doctors’ committee
    B. Ask the Foundation Programme Director if you can move to work at another unit
    C. Ask your consultant if you can support outpatient clinics and theatre sessions
    D. Offer to assist your FY1 colleagues on other busier wards
    E. Inform the Foundation Programme Director that the job should be reviewed to include more learning elements
A

Answer: CDEBA
Rationale: This question is about demonstrating a commitment to professionalism that benefits both yourself and patients. Asking to be scheduled for outpatient clinics and theatre sessions is an active approach to learning which also enables you to contribute and may improve the value of the post for future doctors if that activity becomes part of the culture of the team (C). Whilst offering to assist colleagues on other wards enables you to contribute actively to the hospital in which you are employed, and demonstrates good team working, it doesn’t necessarily contribute to the team to which you have been assigned (D). Option E would enable to the Foundation team to help both you and the clinical team examine whether changes are needed, and is preferable to Option B, which is a less constructive approach (E, B). Taking on additional positions of responsibility, whilst admirable, is something that you should do because you want to, and can fit in around your professional life both in this, and other teams to which you will move (A).

How well did you know this?
1
Not at all
2
3
4
5
Perfectly
12
Q
  1. It has come to the end of your shift, but you have agreed to stay on the ward for another hour due to unforeseen circumstances. A patient, Mr Griffin, is admitted to the ward from the Acute Admissions Unit* (AAU). You notice that Mr Griffin does not have a drug chart or management plan, which should have been completed upon admission.
    Rank in order the appropriateness of the following actions in response to this situation (1= Most appropriate; 5= Least appropriate).
    A. Contact the AAU to discuss Mr Griffin’s management plan and drug chart
    B. Ask the nurse in charge to request the management plan and drug chart from the AAU as soon as possible
    C. Send a message to the FY1 doctor on the next shift stating that Mr Griffin was unfortunately admitted without a drug chart or management plan
    D. Handover to the night shift FY1 doctor to chase the drug chart and management plan
    E. Inform a senior doctor (specialty trainee*) that Mr Griffin was admitted without the correct paperwork
A

Answer: ADBEC
Rationale: This question is about putting the patient first whilst understanding the extent, and boundaries, of your professional responsibilities. Contacting the admitting unit to discuss the patient’s management plan and drug chart allows you to make a clinical judgement as to whether it would be safer for the patient if you stay even later to fill in the paperwork, or whether it would be safe to handover reliably to another doctor (A). Ensuring a proper handover (with documentation) with the night-shift FY1 is acceptable (D). Asking the nurse in charge to contact the admitting unit achieves a similar result, but is less reliable, and since you have not contacted a specific doctor for a medical task, you will have less certainty that it will be acted on (B). A more senior doctor will be in a position to ‘chase up’ the admitting unit to complete the relevant paperwork and drug chart, but they will also have to other duties that require attention from someone at their level (E). There is uncertainty whether the doctor on the next shift will receive and act upon the message, thereby endangering patient safety and patient care, and is therefore the least appropriate (C).

How well did you know this?
1
Not at all
2
3
4
5
Perfectly
13
Q
  1. You are working on a respiratory ward. This ward is attached to a nationally acclaimed academic department. There are posters advertising research projects in all patient care areas. You overhear a patient telling a relative that he is concerned that his personal information will be used in research and made available for all to see.
    Rank in order the appropriateness of the following actions in response to this situation (1= Most appropriate; 5= Least appropriate).
    A. Tell a nurse what you overheard and ask if she can reassure the patient regarding his concerns
    B. Ask the ward manager if communication can be provided to patients explaining that patients’ involvement in research is only carried out with their permission
    C. Ask the patient why he has concerns about the confidentiality of his personal information
    D. Reassure the patient that research will not be carried out using his personal information without seeking his permission
    E. Inform the patient’s relative that his personal information will not be used without his permission
A

Answer: CDBAE
Rationale: This question is about addressing the concerns of a patient and providing reassurance. By asking the patient what his ideas and concerns are you can provide specific and relevant reassurance (C). Giving reassurance without discussion may result in you not gaining a full understanding of the patient’s concerns leading to ongoing worries (D). Option (B) is a useful action in due course, but does not address the immediate issue. Delegating the responsibility for reassurance to someone else is less reliable, as you cannot be sure the task will be completed in a timely fashion (A). Speaking to the relative and not the patient is least acceptable as you are not directly addressing the concerns of the patient (E).

How well did you know this?
1
Not at all
2
3
4
5
Perfectly
14
Q
  1. It is 8am and you have just finished a busy night shift on the Acute Admissions Unit* (AAU). Mr Dean, a patient on your ward with acute renal failure, needs his blood tests to be re-checked in four hours’ time. You approach Gerard, your FY1 colleague, who is starting his shift on your ward. You attempt to hand over the information relating to Mr Dean’s case to ensure that the blood tests are carried out. Gerard says angrily that he has a long list of other patients to see and has just been called to an emergency situation on another ward. He refuses to accept your handover.
    Rank in order the appropriateness of the following actions in response to this situation (1= Most appropriate; 5= Least appropriate).
    A. Stay on the ward to do Mr Dean’s blood tests yourself
    B. Explain to Gerard that he is now responsible for attending to patients on the ward so should accept your handover
    C. Find another appropriate colleague to whom to hand over Mr Dean’s case
    D. Advise Gerard that you will leave detailed instructions regarding Mr Dean’s case in the patient’s clinical records for him to follow up later
    E. Inform a nurse of Mr Dean’s case, asking him or her to find another doctor to conduct the patient’s blood tests

***Difficult

A

Answer: BCDEA
Rationale: This question is about assertiveness and ensuring the completion of tasks. Clarifying responsibilities with your colleagues enhances communication and demonstrates appropriate assertiveness (B). Handing the task to another doctor ensures the task will be completed in the patient’s best interests but would mean another colleague undertaking the Gerard’s tasks in addition to their own (C). Leaving written information about a patient in their clinical record is acceptable, but does not ensure that the task will be completed (it seems an easy option, but does not clearly handover the responsibility) (D).Similarly with Option (E), informing a nurse is less reliable as you have not handed over the task to a specific doctor, and the nurse does not know which doctor to contact (E).Taking the blood test yourself will ensure it is taken, but you will risk errors due to fatigue and not have the opportunity for adequate rest (A).

How well did you know this?
1
Not at all
2
3
4
5
Perfectly
15
Q
  1. You are admitting a patient who does not speak fluent English for an elective operation. He does not have a translator or a relative present. You know from his notes that the patient speaks Urdu. It is apparent that his pain has worsened since his clinic appointment. You ask the patient how long he has been suffering from this pain. The patient appears to understand what you are saying but cannot reply. He is clinically stable.
    Rank in order the appropriateness of the following actions in response to this situation (1= Most appropriate; 5= Least appropriate).
    A. Ask a doctor who speaks Urdu to attempt to communicate with the patient
    B. Continue trying to communicate with the patient to ask about his symptoms
    C. Telephone the NHS language services to obtain a translator
    D. Ask a senior doctor for advice on how to proceed
    E. Telephone the patient’s next of kin to ask about the patient’s medical history and symptoms
A

Answer: ACDEB
Rationale: There are clear NHS guidelines about communicating with patients whose first language is not English; you should always consult these (e.g. on your hospital’s intranet) as an FY1. If an Urdu-speaking doctor was available, asking for assistance would be most appropriate as she/he would be communicating directly and fluently with the patient (A). The second most appropriate action is to telephone (or arrange in advance where known) to obtain a qualified translator (C) although this can take longer than Option (A). This patient’s clinical care is potentially being compromised because of the communication difficulties, particularly if these cannot be addressed, so consulting a senior doctor is appropriate (D). There are potential concerns about involving a patient’s next of kin including confidentiality and also gender or age differences depending on the nature of the patient’s clinical problem (E). Simply continuing without taking any alternative action would not be good clinical care (B).

How well did you know this?
1
Not at all
2
3
4
5
Perfectly
16
Q
  1. The specialty trainee* on your ward, Dr Kitson, is a good friend of yours. She has just sent you a text saying she is running 30 minutes late for work and asks you to cover for her. One of the patients on the ward, Mr Bradley, informs you that Dr Kitson was supposed to be discharging him first thing that morning and it is now 9am. He explains that it is urgent he gets to work by 10am and it is a 45 minute journey to get there.
    Rank in order the appropriateness of the following actions in response to this situation (1= Most appropriate; 5= Least appropriate).
    A. Sign Mr Bradley’s discharge paperwork yourself
    B. Explain to Mr Bradley that Dr Kitson has been delayed so he may want to contact his work and let them know the situation
    C. Contact Dr Kitson and find out whether she can give verbal approval to the discharge
    D. Find another senior colleague in your team to review and discharge Mr Bradley
    E. Offer Mr Bradley the option of signing a self-discharge form

*Difficult

A

Answer: DCBEA
Rationale: This question is trying to ascertain your decision making as a professional who puts the patient first. An alternative senior colleague in the team who would be familiar with Mr Bradley’s case would be the most appropriate person to authorise discharge (D). If senior supervision is not available on site then it would be acceptable to communicate with a senior colleague who knows the patient well and, if safe, facilitate discharge through supervision by telephone with the process reviewed face to face as soon as possible (C). Good communication with patients and keeping them informed at all times is an important duty, however this does not address the time sensitivity in Mr Bradley’s work situation as it does not attempt to prioritise the discharge (B). If safe supervised discharge cannot be achieved by any of the above methods, then the patient should be advised of their right to self-discharge but must be fully informed of the potential risks (E). FY1 doctors should never discharge patients without close supervision so Option (A) is inappropriate.

How well did you know this?
1
Not at all
2
3
4
5
Perfectly
17
Q
  1. A patient, Mrs Mathews, has been admitted for investigation of abdominal pain, which her husband is aware of. You are asked to take a telephone call from him (Mr Mathews), who is asking for an update on his wife’s condition. You have just found out from a urine test that Mrs Mathews is pregnant.
    Rank in order the appropriateness of the following actions in response to this situation (1= Most appropriate; 5= Least appropriate).
    A. Tell Mr Mathews that he will need to speak to Mrs Mathews directly about her condition
    B. Tell Mr Mathews that you would like to obtain Mrs Mathews’ permission to speak to him first
    C. Tell Mr Mathews that you would like to discuss Mrs Mathews’ case with a senior colleague before speaking with him
    D. Tell Mr Mathews that you are currently investigating Mrs Mathews’ abdominal pain
    E. Inform Mr Mathews that Mrs Mathews has had a urine test with a positive result
A

Answer: BDACE
Rationale: This question is about patient confidentiality and your duty of care towards your patient while preserving their confidentiality and maintaining good communication with the patient’s family. Informing the patient’s family member that you need to seek the patient’s permission before divulging any clinical information (B) is the most preferred option. Sharing only the information that the family member is aware of (abdominal pain) would buy you some time and you will not have divulged any confidential information with her husband hence (D) is the next option. The patient’s husband is likely to be under stress and asking him to speak directly to his wife to get information is likely to add further stress which makes option (A) less preferable. Similarly saying that you need to discuss the case with your senior (C) is evasive and doesn’t establish patient consent to discuss the results. Option (E) breaks patient confidentiality hence that is the least desirable option.

How well did you know this?
1
Not at all
2
3
4
5
Perfectly
18
Q
  1. You are on the ward round with your consultant and attend to a patient who is complaining of a severe headache and neck stiffness. Before the consultation has finished, a nurse interrupts to inform the consultant that he is needed urgently to see another patient. The consultant asks you to conduct a lumbar puncture whilst he is away. You have not done or observed this procedure before.
    Rank in order the appropriateness of the following actions in response to this situation (1= Most appropriate; 5= Least appropriate).
    A. Conduct the procedure to the best of your ability
    B. Find another colleague to conduct the procedure whilst you observe
    C. Telephone the neurology specialty trainee* for advice on how to conduct the procedure
    D. Inform the consultant, away from the patient, that you have not conducted this procedure before
    E. Find a senior colleague to supervise you conducting the procedure
A

Answer: DBECA
Rationale: This question is about realising your limitations, seeking senior advice appropriately and assessing your ability to safely and responsibly manage the patient. The preferred conduct would be to inform the consultant responsible for the patient/supervising consultant and seek advice (D). The next best option is to seek advice from another senior colleague and observe the procedure first before performing one (B). Option (E) is preferable to (C) and (A) as you have a senior colleague directly observing you who can intervene if needed without the patient coming to any harm. Option (C) is preferable to (A) as you are getting some advice regarding how to do the procedure before doing it. Option (A) is the least appropriate because this behaviour is potentially dangerous.

How well did you know this?
1
Not at all
2
3
4
5
Perfectly
19
Q
  1. You are checking drugs chart on a general medical ward and you notice that the diabetes specialty trainee* has prescribed double the dose of a tablet for a patient with diabetes. You are aware that sometimes double the dose of this tablet is given to patients. The patient is stable and the specialty trainee is due on the ward in a few hrs.
    Rank in order the appropriateness of the following actions in response to this situation (1= Most appropriate; 5= Least appropriate).
    A. Assume the dosage is correct as you know that sometimes the dose is doubled
    B. Call the specialty trainee to check the dosage with him
    C. Change the dose to the normal amount given
    D. Check with the ward pharmacist whether she is aware of the double dosage for this patient
    E. Check the dosage with the specialty trainee when he comes onto the ward
A

Answer: BDECA
Rationale: This question is assessing how you manage issues of patient safety and how you maintain working relationships. The preferred conduct would be to contact the diabetes registrar/specialty trainee immediately and clarify the reason for the double dose. This behaviour is likely to result in the safe, simple and rapid resolution of the problem, highlight your own clinical vigilance and maintain an amicable relationship between you and the other team members. This would therefore provide a learning opportunity for you (B). The next best option is to seek advice from the pharmacist. If the pharmacist recommended the lower dose, the prescription should not be given without consulting with a senior medical team member (D). Option (E) is preferable to (C) and (A) as it will determine the correct dose, but with a time delay. Option (C) is preferable to (A) as the double dose could cause harm to the patient and the patient is apparently receiving the normal dose without major immediate problems. Option (A) is the least appropriate action as the double dose could cause more harm to the patient if the patient was not actually intended to have the double dose.

How well did you know this?
1
Not at all
2
3
4
5
Perfectly
20
Q
  1. Albert, a 70 year old patient, was admitted mid-morning to the General Medical ward where you are working. Albert was recently diagnosed with a brain tumour and has come back to hospital for further tests. When he was admitted, you advised Albert and his family that an MRI* scan would be arranged within a few hours. The radiology department contacts you to inform you that Albert’s scan will not be performed until tomorrow morning, as a result of urgent cases needing attention this afternoon. You inform Albert’s family of the delay and they react angrily towards you.
    Rank in order the appropriateness of the following actions in response to this situation (1= Most appropriate; 5= Least appropriate).
    A. Inform Albert and his family that the delay is the responsibility of the radiology department
    B. Apologise for the delay, listening to the family’s concerns
    C. Explain the clinical need for other scans to be conducted before Albert’s scan
    D. Inform Albert’s family about the formal complaint procedure
    E. Advise Albert and his family that the information that you gave them earlier was accurate at the time
A

Answer: BCEDA
Rationale: It is most appropriate to apologise as Albert’s family is worried about him and upset that the scan will not take place as quickly as the family was originally told (B). Explaining the cause of delay may help relatives to understand the rationale behind it, but the first step is to apologise and listen to their concerns (C). Arrangements often change quickly in a hospital environment and it may be helpful to tell relatives that when you gave them this information, it was correct (E). It is their right to complain but this option is less appropriate because they may be satisfied if you take time to listen and explain things to them (D).It is not fair to blame the radiology department as they are responding to clinical need as required and this is therefore the least appropriate thing to do (A).

How well did you know this?
1
Not at all
2
3
4
5
Perfectly
21
Q
  1. You and another FY1, Katrina, are working together as part of a surgical team. The ward is very busy and you are taking a blood test from a patient. You notice that Katrina has left the ward without telling you. A nurse tells you that she has gone to assist a surgeon in theatre. Katrina has had her bleeps* redirected to you, which has left you with a very heavy workload.
    Rank in order the appropriateness of the following actions in response to this situation (1= Most appropriate; 5= Least appropriate).
    A. Contact the operating theatre to request that Katrina returns to the ward
    B. Ask Katrina, when she returns to the ward, to speak to you in future when she needs to leave the ward
    C. Tell the senior doctor on the ward that Katrina has left without informing you
    D. Report Katrina to her line manager for leaving the ward
    E. Write a list of the jobs that have arisen in Katrina’s absence so that she can complete them when she returns to the ward
A

Answer: ABCED
Rationale: It is most appropriate to contact Katrina and ask her to return to the ward. She has not discussed this with you and has left you with an unmanageable workload (A). To address the situation in a longer term, it is appropriate to ask that she hands over to you if she is going to leave the ward especially when the ward is pressured. However, she may have been asked by a senior doctor to go to theatre (B). It would be better to speak to Katrina about this directly before informing senior doctor (C). Jobs will need to be prioritised on clinical need and completed as such (E). Referring straight away to Katrina’s line manager for an isolated incident is not appropriate at this stage, as initially you should take the responsibility for finding out why is has happened and take some action before escalation (D).

How well did you know this?
1
Not at all
2
3
4
5
Perfectly
22
Q
  1. You are on your way to add a patient to an emergency theatre list urgently when a nurse approaches you and says that he is concerned about one of your patients, Mr Benn. Mr Benn’s catheter is showing a very low urine output following his surgery earlier that morning. You had already checked on Mr Benn during your ward round two hours previously and had seen that his urine output was adequate given his body size.
    Rank in order the appropriateness of the following actions in response to this situation (1= Most appropriate; 5= Least appropriate).
    A. Reassure the nurse that you reviewed Mr Benn’s urine output on the ward round earlier that morning
    B. Go straight to Mr Benn to review his clinical condition
    C. Explain that you need to add a patient to an emergency theatre list urgently, asking the nurse to find another member of the team to review Mr Benn
    D. Ask the nurse to record vital signs and tell him that you will review Mr Benn once you have added your patient to the emergency theatre list
    E. Ask the nurse whether Mr Benn’s urine output has changed since the ward round this morning
A

Answer: EDBCA
Rationale: This is about prioritisation and delegation. Option (E) is the preferred option as it can provide more detail about recent changes. Option (D) is acceptable as urine output does not change suddenly and you are asking her to gather more data in the interim whilst you deal with the theatre list which could be crucial. Option (B) appears correct but you are automatically assuming that this is more important than completing your current task. Option (C) is less appropriate as you are putting responsibility of your patient on to another doctor via a nurse. Option (A) is least appropriate as false reassurance is then given.

How well did you know this?
1
Not at all
2
3
4
5
Perfectly
23
Q
  1. You are working on a medical ward. You notice that one of your FY1 colleagues on the ward, Bashar, is not completing his assigned tasks. You often find him in the staff room on his mobile phone when he should be completing his tasks on the ward.
    Rank in order the appropriateness of the following actions in response to this situation (1= Most appropriate; 5= Least appropriate).
    A. Accept that it is not your place to intervene
    B. Ensure that you cover any tasks Bashar fails to complete
    C. Discuss this with your educational supervisor* at the end of the placement
    D. Ask Bashar why he is often in the staff room rather than on the ward
    E. Inform a senior colleague of your concerns

**

A

Answer: DEBCA
Rationale: This is about supporting your colleague in difficulty. Option (D) is most appropriate as you have given Bashar a chance to explain and also you find out the real reason giving you an opportunity to rectify and support your colleague in difficulty. Option (E) is less appropriate than (D) in the first instance but this will be your second step as seniors need to be aware of the problems early to be able to address them in time. Patient safety has to be placed above all and their management should not suffer and hence Option (B) is more appropriate than (C) and (C) is less appropriate as it is almost similar to (A) whereby you have waited for the entire four months to come and inform your own educational supervisor who may not have any dealings with Bashar. Option (A) is least appropriate as it means you are not raising alarm when things are going wrong.

How well did you know this?
1
Not at all
2
3
4
5
Perfectly
24
Q
  1. You are an FY1 working on a general medical ward. As you return to the ward from your break you overhear your FY1 colleague, Clare, speaking to one of the healthcare assistants, Melissa, in the corridor. Clare tells Melissa angrily that she needs to improve her skills if she is ever to be any good at her job. Melissa looks visibly upset, apologises to Clare then walks away. You have witnessed Clare talking in a similar way to other colleagues in the past.
    Rank in order the extent to which you agree with the following statements in this situation (1= Most agree with; 5= Least agree with).
    A. Melissa needs to learn to accept feedback from other members of the team
    B. It is not your responsibility to speak to Clare about her behaviour
    C. Clare should not be speaking to members of staff in this manner
    D. Melissa should inform a senior colleague if she has been upset by Clare
    E. Clare needs to be held accountable for her behaviour
A

Answer: CEDAB
Rationale: This situation is a familiar one to doctors working in teams and busy, stressful clinical areas. The first step is to determine your reaction to this event and it is clear that your colleague should not be speaking to a member of the team in this manner (C). In doing this, you form a judgement on the accountabilities for unacceptable behaviour (E). The Healthcare Assistant also has responsibilities as Clare may (and has been observed doing so) replicate this behaviour with other members of the team so escalation by Melissa is appropriate (D). Learning to accept feedback is important, but this is not central to this scenario (A). It is clearly unacceptable for you to do nothing (B), so this is the statement that you should least agree with.

How well did you know this?
1
Not at all
2
3
4
5
Perfectly
25
Q
  1. One of the FY1 doctors working in your team, Haman, had a seizure whilst at a social event that you attended a few days ago. You have known him for some time and are aware that he is on medication for epilepsy. Today another FY1 colleague, who also knows about Haman’s medical condition, tells you that she saw Haman driving to work this morning.
    Rank in order the appropriateness of the following actions in response to this situation (1= Most appropriate; 5= Least appropriate).
    A. Advise your FY1 colleague to speak to Haman about what she observed
    B. Advise your FY1 colleague to seek advice from a senior colleague
    C. Ask Haman if he has been driving after he has had a seizure
    D. Suggest to your colleague that she reports Haman to his consultant
    E. Suggest to your colleague that it is Haman’s decision whether he feels safe to drive
A

Answer: ABCDE
Rationale: As your colleague saw Haman driving it is the colleague’s duty to act. They have directly witnessed the driving and therefore are in a better position to describe what they saw. In the first instance it is better that Haman is given the opportunity to act and this is why Option (A) is a better answer than (B). Option (C) also gives Haman the opportunity to act and is preferable to (D) escalating to the consultant immediately. Option (E) is incorrect and therefore the least correct answer.

How well did you know this?
1
Not at all
2
3
4
5
Perfectly
26
Q
  1. You are on duty at night and see a patient, Mrs Penn. Mrs Penn has developed sudden shortness of breath and signs consistent with acute pulmonary oedema. You have managed this condition successfully before. However, the nurse in charge of the ward wants you to call the specialty trainee* to come to see Mrs Penn. You are aware that the specialty trainee is currently in the Emergency Department caring for a sick patient.
    Rank in order the appropriateness of the following actions in response to this situation (1= Most appropriate; 5= Least appropriate).
    A. Explain to the nurse that you have managed this condition before and can care for Mrs Penn
    B. Make an initial assessment of the patient, administer appropriate treatment and then inform the specialty trainee
    C. Telephone the specialty trainee in the Emergency Department, explain the situation and your experience and follow his advice about what to do
    D. Tell the nurse that Mrs Penn is your patient and that you will take responsibility for your decisions
    E. Go to the Emergency Department to explain the situation to the specialty trainee in person
A

Answer: BCAED
Rationale: This question is about providing good patient care while working appropriately as a member of a multi-disciplinary team. Initiating treatment and then seeking advice about how to escalate if needed ensures that the patient gets help in a timely fashion (B). Seeking telephone advice from a senior doctor increases the confidence of the nurse in your proposed management and will not cause undue delay (C). Explaining to the nurse the reason you do not need help in initial management may have a similar effect (A). Going to the Emergency Department for advice without initiating treatment may cause a significant delay (E). Appearing arrogant towards the nursing staff is the least appropriate as this hinders multi-disciplinary care (D).

How well did you know this?
1
Not at all
2
3
4
5
Perfectly
27
Q
  1. Mr Farmer has been a patient on the ward for six months; he has a tracheostomy and he breathes with the aid of a ventilator following a traumatic brain injury. As you make your rounds, you notice Mr Farmer appears to be experiencing breathing problems. Both the consultant and specialty trainee* are dealing with a patient on the neighbouring ward. This is your first week as an FY1 and you have not yet attended a potentially critically unwell patient by yourself.
    Rank in order the appropriateness of the following actions in response to this situation (1= Most appropriate; 5= Least appropriate).
    A. Call the crash team to attend to Mr Farmer as a matter of urgency
    B. Seek advice from the physiotherapy team who are on the ward and have experience in managing Mr Farmer’s case
    C. Contact the specialty trainee to discuss Mr Farmer’s symptoms
    D. Ask the ward nurse to fully assess Mr Farmer’s status with you immediately
    E. Ask the consultant to return to your ward straight away to attend to Mr Farmer

DCBEA

A

Answer: DCBEA
Rationale: This question is assessing your ability to make appropriate decisions in a pressurised situation. It is important to assess Mr Farmer’s status immediately. The ward nurse is most likely to be the health professional available to help and have the skills, knowledge and ability to access help if needed. It is important not to ‘go it alone’ if possible as help is likely to be required (D). Assessing the status of the patient should be your immediate priority and discussion with a senior colleague (C) could help reach an outcome for the patient. It can be important to have wider team involvement and informing them of patient progress is important (B). However, this would not be an immediate action and is less direct than Options D and C. Consultant return may not be appropriate until the patient is properly assessed (E). Crash teams should only be called in the case of arrest or emergency, doing otherwise could put other patients’ lives at risk and is therefore the least appropriate option (A).

How well did you know this?
1
Not at all
2
3
4
5
Perfectly
28
Q
  1. You are working on an elderly care ward. During the ward round, your consultant asks you to request a CT* scan for Mrs Roberts. You overhear the specialty trainee* saying to another colleague that there is no indication that a CT scan is needed for Mrs Roberts and that it is inappropriate to request one. The consultant does not hear the specialty trainee’s comments.
    Rank in order the appropriateness of the following actions in response to this situation (1= Most appropriate; 5= Least appropriate).
    A. Request the CT scan, as asked by your consultant
    B. Discuss the case and difference of opinion with the consultant radiologist
    C. Discuss with your consultant the reasons for the CT scan
    D. Tell your consultant that the specialty trainee has said that the CT scan is not needed and that the request is inappropriate
    E. Suggest to your specialty trainee that if he thinks the scan is not needed he should raise this with the consultant

*
CEADB

A

Answer: CEABD
Rationale: This question is about ensuring good patient care while dealing with conflict within a team. A discussion about the reasons for the CT scan will mean that the request form can be completed accurately and the most information gained from the scan. It will also allow differing views within the team to be explored (C). A discussion of the different viewpoints initiated by the registrar/ specialty trainee (E) will allow everyone to learn but may be more confrontational than (C). The consultant retains overall responsibility for the patient and so if it is not possible to discuss the reasons for different viewpoints, it is appropriate to follow the consultant’s advice (A). Discussing the case with the radiologist is likely to be beneficial for patient management and your learning but it would be better to understand the reasons for the difference in opinion by discussing with your own team rather than involving another consultant (B). Option (D) is the least appropriate because it gives the appearance of telling tales and may stir up conflict within the team instead of using the disagreement as an opportunity to learn.

How well did you know this?
1
Not at all
2
3
4
5
Perfectly
29
Q
  1. During a consultant ward round, you see a patient who needs to undergo a minor operation. The consultant asks the specialty trainee* to obtain consent, which he does. Later the nurse tells you that she is concerned because the patient does not seem to understand fully what is happening to him, although he is aware he is going to theatre.
    Rank in order the appropriateness of the following actions in response to this situation (1= Most appropriate; 5= Least appropriate).
    A. Check the patient’s understanding of the operation
    B. Advise the patient to withdraw his consent until he has further details of the operation
    C. Ask the nurse to contact the specialty trainee to speak to the patient
    D. Inform your consultant that there may be concerns over the patient’s consent
    E. Inform the specialty trainee that the patient would like further clarification of the operation

**
ACDEB

A

Answer: AEDCB
Rationale: This question involves knowledge of the mental capacity act: in brief to give informed consent the patient should be able to understand the information, retain the information, use or weigh the information and be able to communicate the information. By checking the patient’s understanding of the operation (A) you could establish their level of understanding of the operation, its associated risks and any alternative treatment options – and by doing so yourself, you know for sure that this will be completed. It would be reasonable to relay the concerns of the nurse to the person who took the consent (E) as they may need to review and rephrase their explanation so that the patient understands. The consultant has the ultimate responsibility for the care of the patient so any concerns about consent should be relayed to them (D) if they cannot be resolved. If concerns have been relayed to you then you should take responsibility for resolving them or passing them up the chain of command yourself, not asking the nurse to do this for you (C). It would be inappropriate for you to give this advice to the patient (B) without making an attempt to resolve the issues.

30
Q
  1. A locum senior doctor* has asked you to prescribe a drug for a patient, Mr Singh. You have seen on Mr Singh’s charts that he is allergic to a similar drug and you are concerned that a reaction may occur if you prescribe the suggested drug. The locum doctor does not know the patients on the ward well and recently reacted angrily when another FY1 on the ward questioned one of his decisions.
    Rank in order the appropriateness of the following actions in response to this situation (1= Most appropriate; 5= Least appropriate).
    A. Speak with the on-call pharmacist about whether you should prescribe the drug to Mr Singh
    B. Explain to the locum doctor that you are concerned about prescribing the drug to Mr Singh
    C. Ask another senior doctor whether it is appropriate to prescribe the drug to Mr Singh
    D. Prescribe the drug to Mr Singh, as requested by the locum doctor
    E. Speak about your concerns with the nurse who would administer the drug to Mr Singh

**
BAECD

A

Answer: BAECD
Rationale: This question is about acting in the best interests of the patient and patient safety. If you have concerns regarding a drug you have been asked to prescribe then you should highlight it to the doctor who has asked you to prescribe it (B). If you are unsure if there is a reaction then speaking to the on-call pharmacist is a sensible course of action as they can offer independent advice (A). The nurse also has a responsibility, as they will be giving the drug to Mr Singh and needs to know about your concerns (E). You may find that experienced nurses will bring concerns regarding prescribing to your attention. Talking to another senior doctor about this would also be a reasonable course of action but not as good as speaking to the doctor concerned and introduces another opinion (C). It would be unprofessional and unsafe of you to prescribe a drug that you are worried the patient may be allergic to (D).

31
Q
  1. It is 8am and you are beginning a New Year’s Day shift. A fellow FY1 colleague has called in sick for the same shift; stating that she has food poisoning. The following day you learn that your absent colleague had posted pictures on a social networking site from a New Year’s Eve party that she had attended the night before her shift.
    Rank in order the appropriateness of the following actions in response to this situation (1= Most appropriate; 5= Least appropriate).
    A. Make other colleagues on the rota aware of the photos from the party
    B. Suggest to your FY1 colleague that she remove the photos from the social networking site
    C. Seek advice from another FY1 colleague
    D. Ask your colleague for an explanation of why she called in sick the day after a party
    E. Alert a senior colleague to the photos on the social networking site

DCBAE
*

A

Answer: DCBEA
Rationale: This question is assessing your ability to work effectively in a team and act in a professional manner. Respect for colleagues is important and there may well be a good explanation for her absence. Indeed she may have got food poisoning at the party; therefore asking your colleague for an explanation is an appropriate first action (D). Seeking advice from colleagues on all aspects of professional life is good practice (C). Doctors should be careful with the use of social networking sites as they are open to the public and can lead to impressions about a doctor’s fitness to practise e.g. if apparently intoxicated the night before a shift are they fit to work? Removing the photos would be wise (B). Informing a senior colleague would only be the correct action if there was no adequate explanation forthcoming, the doctor did not show genuine remorse and learning or there were on-going concerns regarding patient safety (E). Advising other colleagues of the photos serves no purpose but to embarrass the individual and is not acceptable (A).
Ideal rank Applicant rank 1 Applicant rank 2 Applicant rank 3 Applicant rank

32
Q
  1. You are working on a care of the elderly ward. Mrs Singh is dying and her family have requested a single room to provide her with some privacy. Another patient, Mr Green is currently in a single room but is well enough to be moved to the four-bed ward. All other single rooms are occupied with patients that cannot be moved. A nurse has advised Mr Green that he is to be moved out of the single room but he has refused as he says he feels uncomfortable being in a ward with other patients. Mr Green’s daughter is a medico-legal solicitor.
    Rank in order the importance of the following considerations in the management of this situation (1= Most important; 5= Least important).
    A. Mrs Singh and her family have a right to privacy
    B. That another single room is unlikely to become available in the near future
    C. That Mr Green’s daughter is a medico-legal solicitor
    D. That Mr Green had the room first
    E. Mr Green’s reasons for wanting to stay in the single room

ABEDC

A

Answer: ABEDC
Rationale: This question is all about the most appropriate allocation of resources on the ward. Maintaining the dignity and privacy of a patient is always a priority and promoting this is the focus of the question for both patients involved. Mrs Sobic has a greater medical and social need and recognising her right to privacy is the most important consideration (A). If resources permit then avoiding conflict and minimising movement of patients takes precedence, but there is no time to wait for this possibility (B). Having established the need for a single room for Mrs Sobic and that Mr Green, from a medical perspective, does not require it, you still need to consider Mr Green’s personal reasons for also wanting a single room (E). Option (D) is important as a general principle but weighing the needs of each patient is more important. Resource allocation and treatment is allocated based on clinical need, not on the social status of a patient or fear of legal repercussions, and the daughter’s profession should have no bearing on your considerations (C).

33
Q
  1. You work on a busy ward with an FY1 colleague. There are many tasks to be completed and the specialty trainee* has said that you and your colleague can go home once the tasks are complete. You have nearly completed your tasks when you are called to another ward on the other side of the hospital. Your FY1 colleague suggests that you call your ward when you are finished, because there will be no need to return to the ward if all the tasks are complete. When you are finished, you telephone your ward but get no answer.
    Rank in order the appropriateness of the following actions in response to this situation (1= Most appropriate; 5= Least appropriate).
    A. Continue to telephone your ward until someone answers
    B. Ask a nurse to contact your ward to say that you have gone home
    C. Send a text message to your FY1 colleague, asking her to call you back
    D. Return to your ward
    E. Leave the hospital and go home

*
DACBE

A

Answer: DABCE
Rationale: This question is about practicalities of safe hospital working. Particularly ensuring the clearest form of communication possible, that ward jobs are not missed and keeping patients under your care safe and uncompromised when you leave. Having failed to obtain an answer from the ward by phone, going to the ward would be the best way of ensuring that outstanding tasks are completed. It also helps to verify whether the lack of an answer suggests that the ward is busy and may need support (D). Continuing to telephone is a less efficient way of obtaining the same result as Option (D) and does not consider the reasons why the ward may not be answering ie an emergency on the ward (A). Asking the nurse does not leave a chance for feedback on outstanding jobs but ensures that someone knows you are no longer available – it is slightly better than leaving without telling anyone in this scenario (B). A text message is a much less certain way of communicating (C). The last option does not ensure that you have safely left your patients and that you still have no outstanding jobs (E).

34
Q
  1. Your consultant has to attend to a patient on another ward. In her absence she asks you to liaise with the radiology department to arrange an urgent CT* scan for Mrs Lewis. You request to book the test but are contacted by the radiologist a few hours later. He informs you that he has rejected your request on the basis of insufficient information.
    Rank in order the appropriateness of the following actions in response to this situation (1= Most appropriate; 5= Least appropriate).
    A. Send the CT scan request to another radiologist
    B. Ask the radiologist to explain in more detail what was missing from the request
    C. Ask your specialty trainee* to discuss the request with the radiologist
    D. Call your consultant to inform her that the radiologist has rejected the request
    E. Ask your consultant to return to your ward so you can explain the situation

BDCEA
*

A

Answer: BCDEA
Rationale: Patient care is of prime importance and this question assesses your ability to make the right decisions to support this. The most appropriate route to obtaining a scan is via a radiologist, therefore talking to them is the most appropriate action. You are likely to also gain important learning for subsequent patient care in understanding what is missing from the request. This option is also likely to lead to better relations in future (B). Training/patient care involves team working, and asking your registrar/specialty trainee is a reasonable option at this stage (C) although it is less direct than Option (B). A consultant asked for request and therefore also needs to know why it was rejected. It may be worthwhile for them to talk to radiology, though this is not an immediate priority (D). It is inappropriate to ask your consultant to return, it would be better to go and find them or negotiate via phone as to the best use of time for everyone (E). It would be undermining and unprofessional to ask another radiologist; indeed it would be likely to lead to problems in the future and is therefore the least appropriate option (A).

35
Q
  1. Mr Reese has end-stage respiratory failure and needs continuous oxygen therapy. While you are taking an arterial blood gas sample, he confides in you that he knows he is dying and he really wants to die at home. He has not told anyone else about this as he thinks it will upset his family, and the nursing staff who are looking after him so well.
    Rank in order the appropriateness of the following actions in response to this situation (1= Most appropriate; 5= Least appropriate).
    A. Tell Mr Reese that whilst he is on oxygen therapy he will need to stay in hospital
    B. Reassure Mr Reese that the team will take account of his wishes
    C. Discuss his case with the multi-disciplinary team* (MDT)
    D. Discuss with Mr Reese’s family his wish to die at home
    E. Discuss Mr Reese’s home circumstances with his GP*

BCEDA
*

A

Answer: BCEDA
Rationale: This question is focusing on effective communication with patients. Ensuring that patients’ informed wishes are met in relation to their care is central to your approach to patient care and this needs to be communicated to the patient in a reassuring manner even in situations relating to end of life care (B).These wishes should have been sought when addressing the management plan for Mr Reese and once identified the MDT needs to be made aware of them in order to ensure that as far as possible Mr Reese’s views in relation to his end of life care are implemented (C). The management of Mr Reese will require the active involvement of his GP and communication with the GP is therefore of importance (E). Any decision to discuss Mr Reese’s wishes in relation to his end of life care with his family can only be made with the full agreement of Mr Reese (D). It would not be appropriate to give the patient inaccurate information in order to engineer a different medical pathway (A).

36
Q
  1. You and another FY1, Robert, are working together on a hospital ward and are sharing ward tasks. After a couple of weeks, it becomes clear that Robert has been taking most of the quicker and easier tasks, and has been leaving you with longer and more difficult tasks. This has allowed Robert to spend additional time collecting data for an audit that his educational supervisor* has asked him to do.
    Rank in order the appropriateness of the following actions in response to this situation (1= Most appropriate; 5= Least appropriate).
    A. Speak with Robert about your concerns regarding the distribution of work tasks
    B. Discuss the distribution of work tasks with your educational supervisor
    C. Report the unfair distribution of work tasks to Robert’s educational supervisor
    D. Complain to the ward consultant about the allocation of tasks
    E. Discuss the situation with another FY1 colleague
A

Answer: ABEDC
Rationale: This question is all about team working and equity of workload. Discussing directly with your colleague is the most appropriate answer as they may be unaware of your perception. Speaking directly allows your colleague to reflect on their behaviour whilst attempting to resolve this informally (A). Discussion with your educational supervisor allows an independent experienced opinion on how big an issue this is. It would be correct to escalate to your educational supervisor regarding the distribution of work as this may affect your ability to avail of the educational opportunities and has the same effect as Option (A) but doesn’t allow an opportunity for amicable resolution (B). Getting a second independent opinion in matters like this is always helpful and may seem easier, although an FY1 colleague will not have the benefit of experience of your educational supervisor (E). Involving the ward consultant may be appropriate but by simply complaining you are not trying to resolve the issue and work collaboratively with others (D). Reporting the unfair distribution to another educational supervisor about a trainee without first discussing with the trainee is unprofessional and allows no chance of reflection or change of behaviour. It does not demonstrate good team working skills (C).

37
Q
  1. At the end of your shift you ordered a blood test and CT* scan for one of your patients, Mrs Tao, who was complaining of feeling faint and confused following surgery. The investigation results need to be reviewed tonight, otherwise Mrs Tao’s treatment may be delayed. You have just arrived home and realise you forgot to hand over the need to review the investigation results to the FY1 doctor taking over your shift. You have been unable to contact the FY1 taking over directly.
    Rank in order the appropriateness of the following actions in response to this situation (1= Most appropriate; 5= Least appropriate).
    A. Telephone the ward nursing staff and ask them to get the FY1 taking over your shift to look up the investigation results
    B. Contact the on-call specialty trainee* and explain the situation
    C. Go back to the hospital and look up the investigation results yourself
    D. Contact an FY1 colleague working on another ward to ask her to look up the investigation results for you
    E. Review the investigation results first thing in the morning when your shift starts
A

Answer: BADCE
Rationale: This question is all about putting the patient first whilst understanding the extent, and boundaries, of your professional responsibilities. Contacting the on-call registrar/specialty trainee to explain the situation enables you to ensure that the patient will be looked after, with reliable handover of the problem (B). Contacting the ward nursing staff to pass on the message to the FY1 achieves the same result but is less reliable, since you have not handed the task over to a specific doctor for a medical task, and you will have less certainty that it will be acted upon (A). Option A is however preferable to Option D, because you are passing the problem on to a colleague who has no on-going responsibility for this patient (D). Returning to the hospital yourself is reliable and will ensure the patient’s safety, but does involve you breaking in to your own time (C). Delaying review of the investigation results carries risk for the patient, and is therefore the least acceptable (E).

38
Q
  1. A patient with a complex medical history dies on the ward after a prolonged period of investigation and treatment. Although enough is known to be able to complete a death certificate, your consultant is keen to arrange a post-mortem to find out more. He gains the consent of the patient’s family for this. However, shortly afterwards the family speak to you as you are passing on the ward. They tell you they felt coerced into saying ‘yes’ to the post-mortem and are upset about the request.
    Rank in order the appropriateness of the following actions in response to this situation (1= Most appropriate; 5= Least appropriate).
    A. Refer the family’s request back to your consultant and ask him to speak to them again
    B. Ask your consultant for his reasons for requesting the post-mortem
    C. Ask another senior colleague within the team to meet with the family to discuss their concerns
    D. Explore the family’s concerns with them
    E. Reassure the family that post-mortems are standard practice in situations like these
A

Answer: DACBE
Rationale: This question is assessing how you respond to the pressure of a stressful situation and communicate effectively in doing so. This is likely to be a very upsetting time for the family and you should respond sensitively to the needs of the bereaved (D). Whilst the medical profession is always keen to learn from practice, particularly difficult or complex cases, if a medical certificate can be completed, there is no necessity for a post-mortem. However, your consultant may wish to discuss their reasons for requesting such an examination with the family again (A), though this is less likely to address the immediate situation presented. Asking the support and advice from other more senior members of the team may be helpful but is a less direct link to the issue (C). The opportunity to discuss this case and the family’s concerns with your consultant will allow you to understand and learn from this experience but would not address the family’s immediate concerns (B). The scandal at a major children’s hospital (Alder Hey) and subsequent inquiry highlighted the need for both clear reasoning and consent in the post-mortem process and it should not be seen as just something that happens, meaning that describing this as ‘standard practice’ is the least appropriate response (E).

39
Q
  1. You are working on a busy surgical ward and are on your way to speak to a patient on another ward about their elective surgery. Mrs Hill, who is the wife of one of your patients, approaches you. She tells you that her husband was meant to be discharged today but that no-one seems to be doing anything. She is very upset as she had made plans for his discharge and she begins to shout at you. Your consultant told you earlier today that Mr Hill will need to be observed for a further two days before he can be discharged.
    Rank in order the appropriateness of the following actions in response to this situation (1= Most appropriate; 5= Least appropriate).
    A. Ask your specialty trainee* to explain to Mrs Hill why her husband is not being discharged today
    B. Tell Mrs Hill that you will speak to her when you return to the ward
    C. Take Mrs Hill into a side room with a nurse to discuss why her husband is not being discharged today
    D. Explain to both Mrs Hill and her husband why he is not being discharged today
    E. Inform Mrs Hill that you cannot speak with her until she has calmed down
A

Answer: DCABE
Rationale: This question is about professional attitudes to relatives/ carers and good communication skills. As long as the patient on the other ward is not ill then you should apologise for any confusion caused around the discharge plans and explain to Mrs Hill and her husband why the discharge has been cancelled (D). As Mrs Hill is clearly upset and disturbing the other patients it would be appropriate to move the conversation to a quieter place for the discussion, however it would be best to involve directly the patient and his wishes. It is good to have a chaperone present if patients and relatives are upset (C). If the explanation does not resolve the situation and Mrs Hill remains unhappy or you are needed urgently on another ward then it would be appropriate to involve a more senior doctor on your team (A) however, it is less appropriate than Option (C) because finding one can cause a delay. By making Mrs Hill wait she can become more upset and so this would not be appropriate except when you cannot attend to her immediately due to urgent care needs of other patients. This would need to be explained to her (B). Refusing to speak to Mrs Hill until she calms down will not be helpful and further inflame a charged situation. It is likely to cause distress to your patient and others who are on the ward including the staff (E).

40
Q
  1. It is 5pm and you are on-call. A nurse from another ward brings you a prescription chart. She tells you that one of her patients has not opened her bowels for five days and, because of this, is very uncomfortable and disturbing other patients. She asks you to sign a laxative prescription as the FY1 doctor on her ward has just refused to do so.
    Rank in order the appropriateness of the following actions in response to this situation (1= Most appropriate; 5= Least appropriate).
    A. Agree to prescribe a laxative
    B. Attend to the patient and make an assessment
    C. Explain to the nurse that if another doctor has told her that a laxative is not required, then that must be the correct decision
    D. Speak to the other FY1 doctor about why he refused to prescribe a laxative
    E. Consult with your specialty trainee* about whether you should prescribe a laxative
A

Answer: DBECA
Rationale: This question is about ensuring the best possible care for the patient, with appropriate attention to safety. Speaking to the other FY1 doctor about why he refused to prescribe a laxative (D) is likely to provide relevant background information, of which you may have been unaware. It is safe practice to assess the patient yourself (B), in conjunction with reading the patient’s records. However, this may be duplicating work already done by your FY1 colleague. Consulting your registrar (E) is appropriate when in doubt about the correct action to be taken. However, you should, whenever possible, first gather relevant background information to enable the registrar to make a decision. Option (C) assumes, without any evidence, that the other FY1 doctor has made the correct decision. It is your own responsibility to decide the best course of action according to the information available. Agreeing to prescribe a laxative (A) is the least appropriate option, and could be dangerous, for example in the case of intestinal obstruction. Therefore diagnoses other than simple constipation should be ruled out before prescribing a laxative.

41
Q
  1. You work on a ward with a nurse, Suzanne, who is also your friend. Suzanne’s mother, Belinda, is a hospital outpatient, whom you have examined previously. Following a discussion with the consultant, you are aware that the results of Belinda’s recent CT* scan suggest pancreatic cancer. The consultant told you that he has asked Belinda to meet with him later on today to discuss the results and has said that her family may attend. Suzanne approaches you later on in the corridor and asks if the meeting has been scheduled because the results suggest bad news.
    Rank in order the appropriateness of the following actions in response to this situation (1= Most appropriate; 5= Least appropriate).
    A. Explore with Suzanne any anxiety she may have about her mother’s results
    B. Confirm the results, advising Suzanne not to tell her mother before the meeting
    C. Tell Suzanne politely, but clearly, that you cannot tell her the results because of patient confidentiality
    D. Advise Suzanne to wait until the meeting as a senior colleague will be better able to explain the results
    E. Ask Suzanne whether she has her mother’s permission to discuss the results before responding
A

Answer: CDAEB
Rationale: This question is about confidentiality. It is our duty as doctors to respect confidentiality of all our patients. Breaking confidentiality is a serious error, and could lead to disciplinary action by the General Medical Council. Reminding Suzanne of the requirement for confidentiality (C) is the most appropriate option. Explaining that senior input is advisable (D) is an appropriate strategy, as it will be necessary to provide complex information and to answer the questions of the patient and her family. Exploring Suzanne’s anxieties will allow you to empathise with her, but it may potentially lead you into a difficult situation, with the risk of inadvertently breaking confidentiality (A). Option (E) is less appropriate, as you cannot be sure that Suzanne is expressing her mother’s wishes. Option (B) is the least appropriate, as it breaches confidentiality with your patient Belinda, and potentially places Suzanne in a difficult position.

42
Q
  1. A pharmacist approaches you on the ward and queries the dose prescribed to a patient. The dose is twice the normal dosage for that particular drug. You know that the drug was prescribed by the consultant, but do not know the reason why. The pharmacist explains that he has seen the dose prescribed at that level for particular cases previously, but wanted to double check as it is quite unusual. The consultant is currently not on the ward.
    Rank in order the appropriateness of the following actions in response to this situation (1= Most appropriate; 5= Least appropriate).
    A. Telephone the consultant to find out the reason why the dose is higher than normal
    B. Explain to the pharmacist that you do not know why the dose is higher and politely request that he checks with the consultant if he is concerned
    C. Change the dosage to the normal amount given until you are able to discuss it with your consultant
    D. Ask the patient if he knows why he is on a higher dose of the drug than normal
    E. Explain to the pharmacist that you would like to keep the dosage as prescribed by the consultant
A

Answer: ABDCE
Rationale: This question is about putting the patient’s safety first and understanding the responsibility that comes with prescribing. The ultimate responsibility for any drug or device lies with the prescriber/user and therefore any queries should be posed directly to this individual. Rather than being worried about what may appear to be ‘questioning’ a senior/specialist decision, discussing with the consultant ensures that the junior doctor is clear about the drug indication and patient management plan, and ultimately ensures that patient care is not comprised in any way (A). Prescribing decisions are complex and certain drugs are only used routinely by specialists. It is therefore entirely appropriate to check things more than once if prescribers or other members of the multi-disciplinary team (MDT)* have concerns about a prescription. The pharmacist is more likely to have an understanding of medication protocols and it is appropriate for him to check again if there are concerns (B). Option A is however preferable to option B, because you would be passing the problem on to a colleague who has no formal clinical responsibility for this patient – this lies with the medical team. Speaking with the patient may help to provide more information about the medication and indication, which again juniors may not be aware of but which may have been relayed to the patient. This is not as reliable as discussing with the prescriber, but can provide some additional information if unable to get in touch with the prescriber immediately (D). If there is any concern about a dosage but if it is imperative that a drug is given, it is safer to administer the drug at a dose that is familiar to all in order to minimise the chance of a complication or adverse effect (C). If a member of the MDT raises a concern about patient care, this should be addressed directly. In this situation, it is highly likely that the pharmacist has greater knowledge and experience of drugs than a junior doctor, and the concern should be acknowledged and acted upon. The consultant may have made a prescribing error, which has been identified by the pharmacist. Not checking the prescription could potentially carry a significant risk for the patient and is therefore the least acceptable (E).

43
Q
  1. You are working on the ward with a nurse, Penny, when you observe that she is not completing hygiene procedures correctly. You have seen Penny check dressings without putting on gloves and neglecting to use hand disinfectant before entering the ward. Penny is a senior nurse with many years of experience.
    Rank in order the importance of the following considerations in the management of this situation (1= Most important; 5= Least important).
    A. The risk to patient safety if correct hygiene procedures are not followed
    B. Patients’ potential anxiety if they notice that she is neglecting to use gloves and hand disinfectant
    C. Penny’s experience working on the ward in comparison to you
    D. That it is the responsibility of the Infection Control* team to monitor and enforce hygiene procedures
    E. The potential for other nurses to follow her example
A

Answer: AEBCD
Rationale: This question is about putting patient care first and making this the first priority for every member of the multi-disciplinary team (MDT)*. Hand washing and infection-control policies are evidence-based and are set to ensure that iatrogenic harm to patients is minimised (A). It is important to always maintain the highest standard of practice as our actions are often emulated by other members of staff and colleagues. We are all role models for our fellow colleagues, many of whom are junior to us, and will follow our example. It is important that they too follow the highest standards of care and do not pick up bad habits, and then carry these forward to their own practice (E). Many policies and standards of care, especially infection control, are well publicised and patients and their relatives are well aware of what should be followed. Our relationship with patients is affected by the level of trust they have with us, as well as our clinical abilities. If a member of the team is not seen to be following basic hygiene precautions, a patient may not feel confident in the team’s abilities with more complex tasks. Also, if the patient acquired an infection or came to harm, this failure to follow a basic measure may be attributed to it (B). Penny is an experienced practitioner and therefore should have a good understanding of the hospital’s protocols. This is particularly the case around infection control which, as a nurse, she plays an integral role in preventing and monitoring. A more junior member of staff may be given more leeway for not being familiar with some hospital protocols early on in their time at a new place of work. It is also important that if a more experienced colleague is doing something incorrectly, it is flagged up and not just ignored due to different levels of seniority or experience (C). Infection control is the responsibility of everyone who enters a clinical area – doctors, nurses, relatives etc. The Infection Control team is an invaluable resource and can give advice, and help to monitor practice, but it is everyone’s responsibility to maintain the highest standards of infection control. (D).

44
Q
  1. You are working on a weekend and are providing cover for two other wards in additional to your usual ward. You are due to finish in 15 minutes. You are contacted by a nurse from one of the other wards, James, who asks you to speak to the relatives of Elizabeth, a patient who has recently been diagnosed with cancer. James tells you that Elizabeth has been informed of the diagnosis, but that the relatives now want to talk to a doctor to discuss the diagnosis and management plan without Elizabeth being present. You do not know Elizabeth, and the regular staff are not working on site this weekend.
    Rank in order the importance of the following considerations in the management of this situation (1= Most important; 5= Least important).
    A. The risk of miscommunication with the family as you are unfamiliar with the patient, her diagnosis and management plan
    B. The potential conflict with James that will be caused if you do not comply with his request
    C. The distress that could be caused to the relatives if they do not have the opportunity to discuss their concerns
    D. The patient’s right to confidentiality
    E. That this conversation is likely to be time-consuming and therefore may mean that you will leave late
A

Answer: DACBE
Rationale: This question is about the various aspects of your responsibility towards the care of your patient, Elizabeth. The requirement for confidentiality (D) must be the highest priority. Speaking directly to Elizabeth, and asking her permission to disclose information to her relatives, will ensure that confidentiality is not inadvertently broken. Since you are not familiar with this patient, there is then a significant potential for miscommunication (A). Reading Elizabeth’s records, and clarifying specific points with her and with James, should reduce this risk. Alleviating distress in your patient’s relatives, wherever possible, (C) is laudable and humane, but your primary duty is to your patient rather than to her relatives. Good working relationships with colleagues are built on complying, wherever appropriate, with reasonable requests (B), but where such requests conflict with your duty to your patient, you must stand firm. Concern about staying over your official duty period is the least important aspect of this scenario (E). As caring professionals, doctors must always be prepared to ‘go the extra mile’ in order to ensure good care. If you have a pressing engagement, then it is your duty to arrange for an appropriate colleague to undertake the care of the patient in your place.

45
Q
  1. You are just finishing a busy shift on the Acute Admissions Unit* (AAU). Your FY1 colleague who is due to replace you for the evening shift leaves a message with the nurse in charge that she will be 15 to 30 minutes late. There is only a 30 minute overlap between your timetables to handover to your colleague. You need to leave on time as you have a social engagement to attend with your partner.
    Rank in order the appropriateness of the following actions in response to this situation (1= Most appropriate; 5= Least appropriate).
    A. Make a list of the patients under your care on the AAU, detailing their outstanding issues, leaving this in the doctors’ office when your shift ends and then leave at the end of your shift
    B. Quickly go around each of the patients on the AAU, leaving an entry in the notes highlighting the major outstanding issues relating to each patient and then leave at the end of your shift
    C. Make a list of patients and outstanding investigations to give to your colleague as soon as she arrives
    D. Ask your specialty trainee* if you can leave a list of your patients and their outstanding issues with him to give to your colleague when she arrives and then leave at the end of your shift
    E. Leave a message for your partner explaining that you will be 30 minutes late
A

Answer: ECDBA
Rationale: This question asks you to demonstrate your commitment to patient care. Although it is not appropriate for trainees to stay for an extensive period of time after their shift ends, or do this on a regular basis, staying an extra 30 minutes on this occasion is important to ensure an effective handover (E). It is more appropriate to provide information directly to your colleague to ensure they receive it (C) and your registrar/specialty trainee could also be able to ensure that your colleague receives the information (D). Leaving lists of information on the end of a bed is less effective and leaving a list in the general doctors’ office is least effective as your colleague is unlikely to know it is there (B, A).

46
Q
  1. On a ward round, the specialty trainee* tells you to write a drug prescription for a patient, Mrs Smith. Before prescribing the drug, you realise that this medicine is contra-indicated with Mrs Smith’s other treatments. The specialty trainee has now left the ward.
    Rank in order the appropriateness of the following actions in response to this situation (1= Most appropriate; 5= Least appropriate).
    A. Write up the drug as requested but omit the start date for the drug until you are able to speak to the specialty trainee
    B. Ask another senior colleague for advice on whether a different drug should be prescribed
    C. Decline to prescribe the drug but write in the patient notes that the drug is contra-indicated for Mrs Smith
    D. Discuss with the ward pharmacist the most appropriate drug to prescribe instead
    E. Try to contact the specialty trainee to inform him of Mrs Smith’s other treatments
A

Answer: EBDCA
Rationale: This question is assessing how you manage issues of patient safety and how you maintain working relationships. The preferred conduct would be to contact the registrar/specialty trainee (E). This behaviour is likely to result in the safe, simple and rapid resolution of the problem. It will also provide feedback to the registrar/specialty trainee, highlight your own clinical vigilance and maintain an amicable relationship between you and the other team members. It may also be that the medication is only relatively contra-indicated in this situation and that the prescription was not an error. This would therefore provide a learning opportunity for you. The next best option is to seek advice from another senior colleague (B). Whilst you are still gaining senior advice, this senior colleague may not be familiar with the patient and their background. Discussing with the ward pharmacist (D) is the next best option. If a pharmacist recommended a different medication, this should not be prescribed without consulting with a senior medical team member. This, however, is pro-active and reasonable behaviour. Whilst declining to prescribe the drug and adding to the patient notes (C) could be considered safe (ie the patient will not be administered the contra-indicated medication), the underlying issue has not been addressed and is less preferable than consulting a pharmacist which suggests that you are actively trying to resolve the problem. Writing up the drug as requested but omitting the date (A) is the least desirable option. This behaviour is potentially dangerous as the medication may well be given in error before you have a chance to speak to your registrar/specialty trainee.

47
Q
  1. You have been working on the surgical team for four weeks. One of the nurses, Jill, has undermined your decisions several times, and has twice called you incompetent in front of patients and staff. You have mainly ignored Jill’s comments, though you did try to speak to her once about your concerns without success. More recently an FY1 colleague told you that you should not allow her to speak to you like that. You have not had feedback from any other team members to indicate that there are any problems with your performance.
    Rank in order the appropriateness of the following actions in response to this situation (1= Most appropriate; 5= Least appropriate).
    A. Continue to ignore Jill’s comments
    B. Inform the nurse in charge about Jill’s comments
    C. Find Jill when she is on a break and ask what her concerns are with you
    D. Inform your consultant about Jill’s comments
    E. Ask other FY1s if they have had similar problems with Jill
A

Answer: DEBCA
Rationale: This question is about working relationships among healthcare professionals. Every member of the multi-disciplinary team has a duty not to undermine colleagues. Such behaviour is destructive to individual professionals, and also destructive to the relationship of trust between patient and carer. Jill is behaving unprofessionally here, and could potentially become subject to disciplinary measures via her professional body, the Nursing and Midwifery Council. You should inform your consultant (D), or Foundation Programme Director, who would then make enquiries, leading to action if deemed appropriate. It is worthwhile to ask other FY1s if they have had similar problems (E), as it would remind them that they, too, should not be expected to suffer such behaviour. You should advise them to discuss it with their consultant, or with the Foundation Programme Director. Informing the nurse in charge (B) is best left to your consultant. Option (C) carries a significant risk that Jill may become antagonistic and verbally hostile; she may potentially make unfounded allegations against you, particularly if you were speaking to her with no witness present. You have already tried unsuccessfully to have an informal conversation. Continuing to ignore Jill’s comments (A) is inappropriate, as she will therefore most probably continue to undermine you, and quite possibly to do the same to others in your position.

48
Q
  1. You are an FY1 doctor working in the Emergency Department. A 48 year old patient presents with an ankle injury. He is intoxicated, loud and demands immediate medical treatment. There are four patients waiting ahead of him. He is upsetting the other patients and at one point, threatens another patient with physical violence.
    Choose the THREE most appropriate actions to take in this situation.
    A. Ask one of the nurses to try and calm the patient down
    B. Call security for assistance
    C. Tell the patient that his behaviour is inappropriate and will not be tolerated
    D. Arrange for the patient to be moved to a side room away from the other patients
    E. Arrange for the patient to be treated as soon as possible
    F. Tell the patient he will not be treated if he continues to behave in this manner
    G. Ensure the other patients are not distressed by the situation
    H. Ask the other patients if they would mind if this patient was treated before them
A

Answer: BCG
Rationale: This question asks you to make decisions in a stressful situation that ensure safety and are communicated effectively. Violence against patients and healthcare professionals is not acceptable. Staff and patients should be protected and hospital security can offer assistance (B). Other staff should not be asked to confront the patient (A) as this may place them at risk, but they can support other patients and minimise their distress (G). The patient should be informed that his behaviour will not be tolerated (C). This should not constitute a threat to his right to treatment (F), but his behaviour should not lead to preferential treatment which may compromise care provided to other patients (D, E, H).

49
Q
  1. At lunch an FY1 colleague bursts into tears. She says she feels she is not coping with the job. She thinks that her consultant is too demanding and the nurses are annoyed because she is not keeping up with the workload.
    Choose the THREE most appropriate actions to take in this situation.
    A. Talk to her about the circumstances of her distress
    B. Suggest she discusses the issues with her specialty trainee*
    C. Advise her to take a few days’ annual leave to rest
    D. Encourage her to get help from her GP*
    E. Offer to go with her to talk to her consultant
    F. Inform her educational supervisor* that she appears to be struggling
    G. Ask other members of the team to be supportive as she is having difficulties coping
    H. Offer to assist with some of her workload
A

Answer: ABE
Rationale: The situation is only described in brief and is likely to be very complicated. It assesses how you work with colleagues and your communication skills. Spending time talking with a colleague can offer much clearer insight as well as often being therapeutic in its own right (A). Similarly, advising talking to a more senior member of the team (B) is likely to bring more support as well as being able to bring a different interpretation of how the FY1 is managing. If, as it seems, the FY1 is experiencing a crisis of confidence, then it cannot simply be left without any agreed way forward, and the person will usually value a colleague’s support in talking to her consultant (E). Taking a few days’ annual leave might bring temporary relief, but will probably not have tackled the problem and returning to work is likely to bring heightened anxiety about coping (C). Getting help from a GP might be appropriate, but will not be immediately accessible and there need to be more immediate actions (D). Asking other members to be supportive removes the responsibility from her (G). Similarly, assisting with her workload is likely to increase the feeling of not coping (H).

50
Q
  1. You are on a busy colorectal team and regularly need to work late to complete all of your tasks. Your FY1 colleague, Jenny, works on the breast team and usually finishes all her jobs by lunchtime. You have 20 new patients this morning and a long list of jobs, including checking blood results. You will need to work late in order to complete all of these tasks. You notice that Jenny is checking her personal e-mails on the ward computer. Your specialty trainee* is busy in theatre.
    Choose the THREE most appropriate actions to take in this situation.
    A. Ask Jenny if she would mind helping you today
    B. Inform your specialty trainee in theatre that you are extremely busy and need some help
    C. Stay at work until you have completed all of your tasks
    D. Talk to your consultant about the unequal workload between you and Jenny
    E. Raise the issue of unequal workloads at the next departmental meeting
    F. Explain to Jenny that it is not appropriate to check personal e-mails at work
    G. Ask a medical student to assist you by requesting x-rays and scans
    H. Hand over the remaining jobs to the on-call team at the end of your shift
A

Answer: ABD
Rationale: This question looks at your team working, communication and planning skills. The first priority is to ensure delivery of care to the patients you have been allocated. Workloads vary and there will be occasions when colleagues should help each other out (A). In addition it is also important that your team is aware of how busy you are and also has an opportunity to assist you if any member is able (B). Long term, however, there seems to be a mismatch in workloads between posts and this needs to be dealt with both in terms of avoiding excessive workloads for some, but also in providing sufficient clinical experience for others (D). It is best to gather more information, however, before raising this in a formal way (E). It should not be necessary for anyone to work for longer than their required hours because of what is essentially an organisational problem, not a medical emergency (C). It is not appropriate to hand over non-emergency jobs to the on-call team, especially if colleagues could have helped during the day (H). Although checking personal emails on a ward computer is not appropriate, it is unnecessarily confrontational to bring this up especially as Jenny may value the chance to gain experience by helping out (F). Students are on the ward to learn and this can legitimately include them being involved in aspects of care. They are not there to prop up a service, however, and should not be given routine tasks that don’t match their educational needs (G).

51
Q
  1. A confused patient has intra-abdominal infection and acute renal failure after a procedure. Instructions to the nursing staff are clearly written in the notes regarding what action to take in response to a change in the patient’s urine output. On the morning ward round, you find these instructions were not followed by the night nursing team and the patient has deteriorated as a consequence. The patient has now received treatment but follow up actions are required.
    Choose the THREE most appropriate actions to take in this situation.
    A. Inform the nurse in charge of the ward of the incident
    B. Explain to the patient that there was an error with the management of her condition
    C. Inform a senior member of the medical team of the incident
    D. Speak to the nurses involved next time you see them about your concerns with their management of the patient
    E. Find out whether the nurses were aware of the instructions regarding changes to the patient’s urine output
    F. Offer to write a protocol formalising team communication
    G. Record your account of the night’s events in the patient’s notes
    H. Ask the nurses to increase the frequency of observations on the patient
A

Answer: ACE
Rationale: This question assesses your ability to work well in a team and communicate effectively. Senior nursing and medical staff need to be aware of the incident so that the incident can be logged, investigated and any action taken (A, C). Instructions had been formulated so it needs clarification as to whether these were communicated (E) but the incident itself would not require a change in these instructions (H). Whilst you should record the events in the patient’s notes (G), this is not an immediate priority. Options (F) and (D) are presupposing the outcome of any discussions so maybe incorrect actions. Option (B) is not immediately necessary and the patient would probably not be able to retain or understand the information as they are confused.

52
Q
  1. You are walking through your ward when you notice that a patient has two tablets in his hands that he is about to consume. The medication was prescribed to him earlier this morning and his case notes clearly state ‘take one tablet twice daily’. The patient in question does not speak English.
    Choose the THREE most appropriate actions to take in this situation.
    A. Arrange for a translator to ask the patient if he knows how he is supposed to take the medication
    B. Prevent him from taking both tablets now
    C. Speak to the doctor who prescribed the medication about the incident
    D. Illustrate on a piece of paper how and when to take the medication
    E. Seek advice on drug dosing from the pharmacist
    F. Inform the nurse in charge about the incident
    G. Find out if his relatives speak good English and can translate the prescription to the patient
    H. Ask the nurse who dispensed the medication to explain why he has more tablets than prescribed
A

Answer: ABH
Rationale: This question looks at communication with patients and your ability to make sensitive decisions. Every patient should be offered a translator wherever possible. It is morally and ethically imperative that patients understand the medical information that is given to them (A). Preventing the patient from taking both tablets now prioritises patient safety. It would be negligent to allow him to take both tablets if you knew this was not what was prescribed (B). By conducting preliminary investigations into the incident and feeding back to the staff involved, you are increasing awareness which may serve to minimise the risk of this happening again (H). Speaking with the doctor who prescribed the medication should be done, however this is not a priority action at this time, particularly as it would appear from the scenario that the prescribing doctor has done nothing wrong (apart from maybe not employing a translator to explain the medication dosing to the patient) (C).Whilst trying to illustrate instructions around taking the pills demonstrates an attempt to communicate with the patient, drawings could be open to misinterpretation and would not be considered medico-legally robust behaviour (D). Involving the pharmacist (E) may be appropriate; however this is not a priority and is not addressing the crux of the issue. Whilst the incident will require reporting, the immediate issues of patient safety, patient explanation and gathering the relevant information to accurately describe the circumstances surrounding the incident are more pressing and required BEFORE speaking to the senior nurse (F). Involving the relatives clearly breaches issues of patient confidentiality (G).

53
Q
  1. You are working on a surgical ward and have been asked by the consultant to do a number of tasks: blood from four patients; chase up results from the ward patients and also from patients in yesterday’s clinic; and to sort out infusion prescriptions for two patients due in this morning. You are approximately half way through these tasks and you are sure that you have a sufficient time during the morning to complete the rest. The consultant bleeps* you to ask you why the tasks have not been completed yet. You have heard from your FY1 colleagues that this consultant has a reputation for giving Foundation doctors lots of tasks to complete within a short period of time.
    Choose the THREE most appropriate actions to take in this situation.
    A. Explain to your consultant that it is unrealistic to expect all of the tasks to be completed by now
    B. Tell the consultant that the tasks will be completed as soon as possible
    C. Explain to the consultant what tasks you have done, and how long they have taken
    D. Ask your FY1 colleagues for assistance with completing the tasks
    E. Inform the Foundation Programme Director* that the consultant is giving junior doctors unrealistic timeframes to complete tasks
    F. Ask the consultant whether he would be able to help you complete some of the tasks
    G. After the tasks have been completed, seek advice from your educational supervisor* about how to approach situations such as these
    H. Try to speed up completion of tasks by deferring some of the paperwork until later
A

Answer: BCG
Rationale: This question is about your professionalism and your communication skills. It is important to communicate with your consultant, making it clear what you have done and to reassure them that you are completing tasks efficiently and will complete them as soon as you are able (B, C). This discussion should be done in a professional way, avoiding confrontation at this point (A). There may, however, be learning points for the future in terms of the prioritisation and completion of multiple tasks and this is always worth discussing with a supervisor (G). Although this consultant has ‘a reputation’, this may or may not be accurate and it is too early to escalate this (E), although this may become necessary later. There is no clinical urgency, so for now, it is not appropriate to ask for help from colleagues simply to appease a consultant (D). It is also important to ensure each task is completed properly (H). Presuming that this consultant does pressurise junior staff excessively and asking them to complete some of the tasks is likely to escalate the situation unnecessarily (F).

54
Q
  1. You have just started your shift. You have not met one of the patients, Mrs Gordon but you know from the handover that she is being treated for a fractured knee and is recovering well. Her husband has noticed that she has lots of bruising around her knee. He angrily approaches you claiming her doctor is not doing her job properly as the bruising is getting worse.
    Choose the THREE most appropriate actions to take in this situation.
    A. Ask a senior colleague to speak to Mr Gordon
    B. Ask the nurse who has been caring for Mrs Gordon to speak with him
    C. Ask Mr Gordon to lower his voice as he is disrupting the other patients
    D. Tell Mr Gordon the other doctor has now finished her shift so you are now taking over responsibility for Mrs Gordon
    E. Try to answer any questions Mr Gordon has
    F. Reassure Mr Gordon that the bruising will be the result of the fractured knee
    G. Set up an appointment for Mr Gordon to meet with the consultant
    H. Re-examine Mrs Gordon’s knee
A

Answer: DEH
Rationale: This question examines your professionalism, communication and empathy. The priorities in this scenario are clearly excluding any new medical pathology (H) and being open, honest and attentive to Mr Gordon’s concerns. Angry patients are often just scared, upset and afraid. Mr Gordon needs to know that you are not his wife’s regular doctor (D). He is therefore much more likely to be understanding if you cannot fully answer all his questions. You should try to answer as many of his questions as you feel able to (E). Asking a senior colleague to speak to him (A) would be the next most appropriate action, however it would be reasonable for you to at least try to speak to him first. Similarly, it may be that in addition to you speaking to him, an appointment with the consultant may be appropriate, but the scenario indicates that he needs a doctor to speak to him immediately. Asking the nursing staff to speak to him (B) is inappropriate. He is clearly concerned about his wife’s medical care (rather than nursing care) so a doctor should speak to him. Not allowing him to speak to a doctor is likely to infuriate him further. As an FY1 you are not going to be able to reassure him that the bruising is simply due to the fracture as there may well be something else going on (F). The patient is likely to need senior review and possibly further investigation before anyone can say for certain that it is purely as a result of the fracture. Asking Mr Gordon to lower his voice (C) is likely to infuriate him further and is therefore going to be detrimental to the situation.

55
Q
  1. You have worked on a ward with another FY1 colleague, Ben, for the last three weeks. You have noticed that Ben seems to avoid writing prescriptions and filling in drug charts. The nurses appear to be getting frustrated with Ben because of this. When you speak to Ben about it, he tells you that he is dyslexic but has not told anyone as he is embarrassed.
    Choose the THREE most appropriate actions to take in this situation.
    A. Make the nursing staff aware of the situation
    B. Check all the drug charts and prescriptions Ben has recently written
    C. Explain to Ben the potential safety risks to patients if he continues to ignore the issue
    D. Discuss the situation with a senior colleague
    E. Recommend to Ben that he raises this with his educational supervisor*
    F. Offer to help Ben with his prescriptions and drug charts
    G. Discuss the situation with your team and ask them to help you monitor Ben’s drug charts and prescriptions
    H. Speak to Occupational Health* (OH) about the best course of action to take
A

Answer: CDE
Rationale: This question asks you to make professional and patient-focused decisions. This is indeed a potential safety issue, although it should have been picked up before applying to the Foundation Programme. Ben really should discuss this with a senior colleague himself and you have a duty to the patients, and to him, to do so yourself, to ensure the issue is raised (C, D, E). Offering to check the charts and support Ben, whilst noble, is not the way forward since this should be undertaken by someone more senior, and doesn’t deal with the problem (B, F). Raising the problem more widely is not your responsibility and needs to be undertaken sensitively once more senior doctors are involved (A, G, H).

56
Q
  1. You have been prescribed codeine for persistent back pain which has become worse in the last few weeks. You have noticed that during shifts you are becoming increasingly tired, finding it difficult to concentrate and your performance, as a result, has been less effective.
    Choose the THREE most appropriate actions to take in this situation.
    A. Ask a colleague to assist with your workload until you finish your codeine prescription
    B. Make an effort to increase the number of breaks during your next shift
    C. Stop taking the codeine immediately
    D. Make an appointment to see your GP*
    E. Seek advice from a specialist consultant about your back pain
    F. Arrange to speak with your specialty trainee* before your next shift and make them aware of your situation
    G. Seek advice from your clinical supervisor* regarding further support
    H. Consider taking some annual leave
A

Answer: DFG
Rationale: This question looks at how you demonstrate commitment to professionalism and self-awareness. The essential problem is that as an FY1 doctor the level of your clinical performance is dropping. This constitutes a risk to the patients you are caring for and will impose a greater workload on your colleagues. In this circumstance you should inform and seek the advice of the senior clinician responsible for your work (G) and alert your colleagues (F). This matter is most likely to be related to your prescribed medicine and you should therefore consult with your GP (D) rather than any other specialist (E). It is not your place to re-allocate workload (A). Increasing the number of breaks is unlikely to improve the situation that is likely to be due to an adverse effect of a drug (B). You should not make any unilateral decisions about your medical treatment (C) and should seek the advice of others (D). You should not be seeking to use your annual leave (H) to compensate for a medical problem.

57
Q
  1. You become aware that one of your FY1 colleagues, Daniel, is consistently not doing his fair share of the ward work. His night shift colleague has told you that he leaves much of the routine work for her and provides poor handover information. However, he is personally very likeable and always performs jobs diligently when directly requested. You know that no-one has broached this with him yet.
    Choose the THREE most appropriate actions to take in this situation.
    A. Discuss Daniel’s behaviour with his clinical supervisor*
    B. Suggest to the nursing staff that they ask Daniel directly to complete the routine work
    C. Explain to Daniel that his behaviour means colleagues have to do extra work and this could impact on patient safety
    D. Bring up the issue of effective handovers at the next team meeting
    E. Ask Daniel if he needs help with his handover
    F. Discuss the situation with your consultant
    G. Ask other staff on the ward if they are experiencing problems with Daniel
    H. Suggest to your night shift colleague that she speaks to Daniel directly about him not completing his tasks
A

Answer: CEH
Rationale: This question assesses how you manage your professional working relationships and with suboptimal conduct of your colleagues. The most appropriate action is to discuss the issues with Daniel himself (C). Alongside this, it would also be appropriate to offer to help him with his handover (E) as you are therefore attempting to remedy the situation. This is acceptable if this is within your capabilities. Asking the other colleague who is being particularly affected by his behaviour to address him personally (H) before going to his seniors/supervisor would also be considered appropriate. Consulting senior medical staff and clinical supervisors (F, A) may well be prudent; however this would be the next step if speaking to him personally was ineffective. The same also goes for gathering further information and opinion on him from the nursing staff (G). Raising effective handovers at the next team meeting is relatively non-specific and not an immediate priority (D). It is inappropriate to ask the nurses to liaise with him directly regarding routine work (B). The situation needs to be properly addressed and, in order for effective and safe patient care, the whole team must be communicating and functioning effectively.

58
Q
  1. You are working in the Emergency Department (ED). Mrs Gersbach, a 65 year old female patient, is admitted with chest pain. This is her fifth attendance with chest pain in the last two weeks. She has been extensively investigated over this time, and a cardiologist has documented that all investigations have been normal and that her pain is not cardiac in origin. Today, nothing on examination or any of the investigations suggest that her pain is a symptom of cardiac disease. There are no other worrying signs or symptoms.
    Choose the THREE most appropriate actions to take in this situation.
    A. Admit Mrs Gersbach to the Acute Admissions Unit* (AAU) for further investigations
    B. Write to Mrs Gersbach’s GP, asking her to dissuade Mrs Gersbach from attending the ED
    C. Explain to Mrs Gersbach that there appears to be nothing wrong with her
    D. Reassure Mrs Gersbach that her pain is definitely not a symptom of cardiac disease
    E. Ask a senior colleague to speak with Mrs Gersbach
    F. Ask Mrs Gersbach if there is anything that she is worried about that might be causing her pain
    G. Arrange an outpatient exercise tolerance
    for Mrs Gersbach
    H. Tell Mrs Gersbach that her attendance at the hospital is not the best use of the ED’s doctors’ and nurses’ time
A

Answer: DEF
Rationale: It is always appropriate to reassure patients (D) who are concerned. It is also appropriate for a senior colleague to speak with Mrs Gersbach (E) in order to provide additional reassurance, and to ensure that there are no new causes for concern in her diagnosis. Eliciting the true cause of repeated attendance (F) will help to direct further management correctly. Since Mrs Gersbach has already been extensively investigated by a cardiologist, options (A) and (G) would not be appropriate. Options (B) and (H) are unnecessarily confrontational, whereas option (C), far from being reassuring, will give the patient the impression that her concerns are not being taken seriously.

59
Q
  1. An FY1 colleague, Lewis, arrives late into work on a frequent basis. You think you may have smelt alcohol on his breath previously, but were not sure. This morning Lewis smells strongly of alcohol and is unsteady when walking.
    Choose the THREE most appropriate actions to take in this situation.
    A. Ask Lewis if he has been drinking alcohol
    B. Tell Lewis that he should go home immediately if he is unfit to work
    C. Inform the consultant in charge of the ward that Lewis smells of alcohol and is unsteady when walking
    D. Seek advice from a senior colleague about how to manage the situation
    E. Explore with Lewis if there are any reasons behind his behaviour
    F. Tell Lewis that if this happens again, you will have to tell the consultant
    G. Tell Lewis to get a cup of coffee
    H. Inform the General Medical Council* (GMC) of your concerns about Lewis
A

Answer: ABD
Rationale: The suspicion that your colleague has been drinking alcohol is strong, and this should be confirmed first-hand (A). Your first concern should be for the patients under his care, so it is essential that he does not remain at work if his judgement and ability are impaired (B). It is also appropriate to ask for senior help and advice in this situation (D). Informing the consultant in charge of the ward is unnecessary (C) unless your colleague insists on remaining at work. Exploring the underlying issues at the time (E) will be fruitless, and is best done in a more appropriate environment by a trained counsellor. Option (F) is unduly confrontational. Option (G) plays down the gravity of the situation, whereas option (H) is unnecessary, as this step is best undertaken at the end of a recognised pathway by a senior colleague.

60
Q
  1. You are in the canteen and are joined by an FY1 colleague, Amadi, who is not on shift. Amadi tells you that his girlfriend was involved in a serious road accident last week and is now in the Intensive Care Unit* (ICU). He tells you that he is very stressed and has barely slept in the last week. He thinks that he is going to have difficulty coping with his weekend shift. He has refused to take compassionate leave as he does not want to leave the ward understaffed.
    Choose the THREE most appropriate actions to take in this situation.
    A. Advise Amadi to see his GP*
    B. Suggest to Amadi that sleeping tablets may help
    C. Advise Amadi to contact the Trust’s counselling service
    D. Advise Amadi to seek time off work from his consultant
    E. Discuss your concerns with Amadi’s educational supervisor*
    F. Advise Amadi to discuss this with his educational supervisor
    G. Offer to go somewhere more private to discuss Amadi’s concerns
    H. Contact the ICU to find out about Amadi’s girlfriend’s health situation
A

Answer: DFG
Rationale: There are two aspects here: the first is the duty of care and compassion towards a colleague in a time of difficulty; the second is a duty of care towards patients in recognising a colleague who is unlikely to be able to fulfil their duties adequately and safely. Option (D) is clearly appropriate in that the situation will impact directly on consultant’s firm and the safety of the patients, and the consultant is most likely to be able to organise cover for the doctor. Option (F) is appropriate because the educational supervisor is most involved with pastoral care of the trainees. Taking Amadi somewhere private to discuss further will help get a full and confidential disclosure of the problems (G). However, a direct action is likely to be required such as D or F. There is insufficient evidence of a clear and present danger to patients to warrant informing a senior without the colleague’s consent – this eliminates option (E). Options (A), (B) and (C) are not inappropriate but do not address the issue of the imminent weekend on-call. (H) is clearly inappropriate as you are not looking after Amadi’s girlfriend so confidentiality of the patient would be breached.

61
Q
  1. You are treating Mrs Taylor for a urinary infection. You receive the antibiotic sensitivity results. You inform your consultant that the E coli causing Mrs Taylor’s urinary infection is sensitive to amoxicillin. As a result, this antibiotic is then started. You realise later that day that you have misread the result and that the E coli is actually resistant to amoxicillin and is sensitive only to ciprofloxacin. Mrs Taylor had no adverse reaction to the antibiotic.
    Choose the THREE most appropriate actions to take in this situation.
    A. Inform your consultant of the correct result
    B. Record the incident as a learning point in your learning portfolio*
    C. Contact microbiology for advice
    D. Tell Mrs Taylor that you have prescribed the incorrect antibiotic
    E. Reassess Mrs Taylor’s vital signs
    F. Change the antibiotic to ciprofloxacin
    G. Repeat the urine specimen culture
    H. Ask your specialty trainee* for advice
A

Answer: ADF
Rationale: This question asks you to make decisions that demonstrate your professionalism. Your consultant might want to know that a change to the management is required, and the patient needs to know that an error (albeit minor) has occurred (A, D). Changing the antibiotic is required to treat the patient (F). Repeating the urine (G) and contacting microbiology (C) will add nothing and, in the absence of an adverse reaction reassessing the patient’s vital signs is unlikely to be of benefit (E). Recording the incident as a learning point is a good idea but is of lower priority (B). If your consultant is available that is the first port of call. Your registrar/specialty trainee would be a reasonable alternative (H).

62
Q
  1. You review a patient on the surgical ward who has had an appendicectomy done earlier on the day. You write a prescription for strong painkillers. The staff nurse challenges your decision and refuses to give the medication to the patient.
    Choose the THREE most appropriate actions to take in this situation.
    A. Instruct the nurse to give the medication to the patient
    B. Discuss with the nurse why she disagrees with the prescription
    C. Ask a senior colleague for advice
    D. Complete a clinical incident form
    E. Cancel the prescription on the nurse’s advice
    F. Arrange to speak to the nurse later to discuss your working relationship
    G. Write in the medical notes that the nurse has declined to give the medication
    H. Review the case again
A

Answer: BCH
Rationale: Ensuring patient safety is key to this scenario. It is important to discuss the nurse’s decision with her as there may be something that you have missed when first reviewing the patient (B). Therefore it would also be important to review the patient again (H). Also relating to this is the importance of respecting the views of colleagues and maintaining working relationships, even if there is disagreement. As there has been a disagreement regarding patient care, it is important to seek advice from a senior colleague (C).

63
Q
  1. You have been approached by an FY1 colleague, James, who has been on shifts with another FY1 doctor, Mark, for the last two weeks. James tells you that Mark has become increasingly careless in monitoring and documenting patient records. On three occasions, James tells you that he has found Mark asleep in the common room whilst on duty. You know Mark very well and have never witnessed such behaviour when you have worked with him previously.
    Choose the THREE most appropriate actions to take in this situation.
    A. Tell James that you have never witnessed such behaviour from Mark
    B. Suggest to James that he speaks to Mark directly about his concerns
    C. Advise James to document his concerns
    D. Inform a senior colleague about what James has told you
    E. Tell James you will speak to Mark about his behaviour
    F. Ask other members of the team whether they have witnessed this behaviour in Mark
    G. Advise James to speak to a senior colleague about his concerns
    H. Ask James whether he has any evidence that patient safety is being compromised
A

Answer: BGH
Rationale: This scenario presents a number of conflicts between professional and personal concerns. James should speak directly to Mark about his concerns (B). James has a professional duty to share his concerns with a senior colleague (G), and must do so immediately if he has evidence of patient safety concerns (H). It will be important to keep a professional oversight and not to judge the observations of James (A, C), or act as a third party between James and Mark (D, E, F).

64
Q
  1. It is the end of your shift and on your way out of the ward you remind one of the nurses that Mr Baker, who has been admitted to the hospital with chest pains, needs his blood taken within the hour for cardiac markers. The nurse tells you that the ward has now got very busy so Mr Baker will have to wait. She is very abrupt in her response to you.
    Choose the THREE most appropriate actions to take in this situation.
    A. Ask the nurse for further details about the other urgent tasks that need to be completed on the ward
    B. Ask the FY1 taking over your shift to take Mr Baker’s blood
    C. Discuss with the nurse in charge the procedure for taking essential bloods at busy times
    D. Insist that the nurse tries to find the time to take Mr Baker’s bloods
    E. Take Mr Baker’s blood yourself
    F. Suggest to the nurse that she considers how she speaks to other members of the team in future
    G. The next day, speak to the nurse privately about the way she spoke to you
    H. Speak to the nurse in charge about the way the nurse spoke to you
A

Answer: BCG
Rationale: This question looks at your communication skills. The first priority is that the patient is looked after, so asking a colleague on duty to do this is appropriate (B). Having said this, the underlying cause needs to be addressed after some information gathering (C), which can then be transmitted to the nurse sensitively at a later time (G). It should not be necessary for anyone to work for longer than their required hours because of what is essentially an organisational problem, not a medical emergency (E). Asking the nurse to take on additional tasks (D) or for more information about the detailed tasks (A) would be inappropriate input to someone else’s job, and options (A), (E) and (F) are counterproductive in time terms. Options (D), (F) and (H) would be confrontational and unwise in the heat of the moment.

65
Q
  1. You are working on a busy medical ward and you have one hour left of your shift. During that time you have to complete paperwork, which will take approximately 45 minutes, and see three patients who may take 15 minutes each. A healthcare assistant asks you to speak to a relative of a patient on the ward, who has called the ward asking to speak to someone.
    Choose the THREE most appropriate actions to take in this situation.
    A. Ask a nurse on the ward to assist you with seeing the patients
    B. Ask the healthcare assistant if he or she can ask a nurse who knows the patient to speak with the relative
    C. Stay late at work to ensure that you complete all of the tasks
    D. Finish the most important clinical tasks and hand over any uncompleted tasks to the next shift
    E. Ask an FY1 colleague to assist you with completing your tasks
    F. Inform a senior doctor that you will be unable to complete all of your tasks during your shift
    G. Agree to speak to the patient’s relative on the telephone
    H. Ask the patient’s relative to call back later to speak to a doctor
A

Answer: BDE
Rationale: The issue here is the recognition of priorities. There is a clear distinction between clinical and operational priorities. Paperwork is part of the latter. Reviewing patients is the priority before handing over. Option (B) is appropriate as it appropriately deflects this new additional demand on your time. Option (D) is clearly appropriate as the clinical tasks should take priority and be finished first. Option (E) is appropriate because two people working in parallel would likely complete all of the tasks in time for handover. Option (A) is an inappropriate imposition on the nursing staff and would likely be refused in any case. Option (C) is inappropriate as a significant proportion of tasks are operational in nature and handing these over would facilitate not overstaying. Option (F) is irrelevant. Seniors do not need to be made aware of incomplete non-urgent tasks; they simply need to be handed over to the contemporary person taking over. It is customary for nursing staff to provide telephone updates to patients. Options (G) and (H) are therefore inappropriate.

66
Q
  1. You are reading some medical magazines in the hospital canteen, whilst on a break. In one magazine, you come across an advertisement for private specialist services bearing the picture and name of Simon, a fellow FY1. You are aware that providing such services is not allowed for FY1 doctors as full registration has not been granted by the GMC* and they can only work in an approved programme and setting. You are aware that Simon is experiencing severe financial difficulties.
    Choose the THREE most appropriate actions to take in this situation.
    A. Suggest to Simon that he tells his educational supervisor* about the advertisement
    B. Suggest to Simon that he stops providing this service immediately
    C. Inform the Foundation Programme Director* about Simon’s advertisement
    D. Discuss with Simon alternative ways for him to make money
    E. Ask fellow FY1s for their opinion on Simon’s actions
    F. Ask Simon whether he is aware that providing specialist services is not allowed as an FY1 doctor
    G. Ask Simon whether anyone is aware that he is advertising these services
    H. Inform Simon that it is unacceptable to place such advertisements
A

Answer: ABF
Rationale: This question looks at dealing with a colleague in difficulty. It is not your role as a fellow junior doctor to deal with this problem on your own and it is important that Simon seeks help and advice from a senior colleague (A). It is important to let Simon know that it would be best to stop providing this service immediately (B) but also to make sure that Simon realises that this practice is not allowed and that his actions are therefore inappropriate (F). You should not escalate this to the Foundation Programme Director as a first point of contact as there is a clear policy that should be followed (C). It is not your role to act as Simon’s senior (H). In the same way it is not appropriate to involve Simon’s peers (E), try to solve Simon’s financial problems yourself (D) or seek to cover up the incident (G).

67
Q
  1. During a ward round, your consultant loses his temper and shouts at the other FY1 doctor on your team for not having ordered blood tests for a patient. This incident is overheard by the patient concerned, the nursing staff and the rest of the medical team.
    Choose the THREE most appropriate actions to take in this situation.
    A. Once away from the bedside suggest to the consultant that the situation felt uncomfortable and you were worried that the patient had been upset
    B. Tell the consultant later in private that you think that his behaviour was inappropriate
    C. Advise your FY1 colleague that he should speak to the consultant about the incident
    D. Discuss with the nursing staff whether this is usual behaviour for the consultant
    E. Advise your FY1 colleague to speak to his educational supervisor*
    F. Apologise to the patient after the ward round
    G. Ask a more senior colleague on the team for advice
    H. Ask your FY1 colleague if he needs any help with his workload
A

Answer: AEG
Rationale: This question deals with communication issues. Patient care is the most important thing and as such you do have a duty to initiate further discussion and this is best done in a non-confrontational manner (A). It is important that the FY1 involved with the issue is provided with someone to discuss the incident with, to facilitate reflection (E) and you may need advice from a senior doctor in the team to help with this difficult situation (G). It is not appropriate for you to have a private conversation with the consultant (B), expect the FY1 to deal with the problem directly with the consultant without a third party (C), to discuss your consultant’s behaviour with the nurses (D) or apologise on behalf of the consultant (F). Whilst you may wish to see if your FY1 colleague is managing his workload, it is not a priority in answering this question (H).

68
Q
  1. You are reviewing one of your patients, Mrs Hobbs, who is on your ward being treated for an infection in her toe. During a routine examination, you notice that in Mrs Hobbs’ drug chart the FY2 has prescribed her penicillin and the administration is due in 45 minutes’ time. You remember your consultant informing you earlier that day that Mrs Hobbs was allergic to penicillin.
    Choose the THREE most appropriate actions to take in this situation.
    A. Inform the FY2 that they have made an error
    B. Cross out the prescription on Mrs Hobbs’ drug chart, dating and initialling the amendment
    C. Tell the nursing staff on duty not to administer penicillin to Mrs Hobbs
    D. Contact your specialty trainee* to confirm what the consultant has said about the penicillin allergy
    E. Inform your consultant about the situation
    F. Ask Mrs Hobbs whether she is allergic to penicillin
    G. Explain to Mrs Hobbs that an incorrect prescription has been made by the FY2
    H. Review Mrs Hobbs notes to try and clarify whether she is allergic to penicillin
A

Answer: BFH
Rationale: In this situation you have spotted a possible prescribing error with the potential to result in a fatality or serious injury. Your first duty is to ensure that this danger is eradicated (B). The next priority is to try and establish the facts of the matter by taking a history from the patient (F) and looking for further evidence in the medical notes (H). While it is always appropriate to acknowledge errors to patients, this probable error did not reach the patient and so little is to be gained from explaining the situation to Mrs Hobbs (G). While it is always important to explore the causes of ‘near misses’ with other members of the team (A, E), this discussion is not a priority at this time. If the correct action is taken with regard to the prescribed medicine (B) there should be no need to give specific instructions to the nursing staff (C). Asking the registrar/ specialty trainee to confirm the consultant’s statement (D) might help, but still leaves room for error as he/she may not have been present or may also have misheard/may not recall correctly.

69
Q
  1. A 45 year old alcoholic is admitted in the afternoon with delirium tremens after stopping drinking two days previously. During the night, you are called to see him as he has become very aggressive and is demanding to be allowed home. As you arrive on the ward he punches one of the nurses. He is confused, shouting and threatening other patients.
    Choose the THREE most appropriate actions to take in this situation.
    A. Prescribe extra sedation for the patient
    B. Ask the nursing staff to call hospital security
    C. Attempt to talk to the patient to try and calm him down
    D. Reassure the other patients in the ward that they are safe
    E. Ask the nursing staff to help you restrain the patient
    F. Ask the nursing staff to call the police
    G. Inform the patient that his behaviour is inappropriate and will not be tolerated
    H. Ensure that the nurse who was punched is not badly injured
A

Answer: BCH
Rationale: This question assesses your ability to cope with pressure and ensure the safety of yourself and other patients. Violence against health professionals (and patients) is not acceptable, but for this patient it is in the context of delirium tremens, in which the patient is confused and agitated, so he would not have insight into his actions. Hence options (G) and (F) are not appropriate. However, an FY1 (or another member of staff) should be protected and hospital security (B) can offer assistance. Even though a patient is confused, they will often calm down if approached in a reasonable manner, reducing the emotion in the situation (C). An FY1 also has a duty to ensure the safety of other staff, so checking on any harm done is important (H). Physically restraining the patient will be difficult and might cause further harm for staff and the patient. It can constitute an assault. Similarly, prescribing extra sedation might exacerbate the confusion and would likely be very difficult to administer without first calming the patient down. Consequently options (A) and (E) are not appropriate. Giving reassurance to patients (D) is important, but in this case, may not be true until the situation is under control, which is the primary focus.

70
Q
  1. You share the responsibility for the patients on the Cardiology ward with your FY1 colleague, Sam. He has previously confided in you that he is finding his FY1 role very difficult. It is the end of Sam’s shift and he informs you that he has been unable to complete all his tasks and once again passes them on to you to finish as you are on a late shift.
    Choose the THREE most appropriate actions to take in this situation.
    A. Speak to your fellow FY1s to see whether they think that his conduct is acceptable
    B. Suggest to Sam that he may want to discuss his difficulties with his clinical supervisor*
    C. Advise Sam to inform his consultant that his workload is excessive and unachievable
    D. Suggest to Sam that, on this occasion, you share the urgent jobs that need to be done with another colleague
    E. Delegate some of the tasks to your other FY1 colleagues
    F. Discuss with Sam what he is finding difficult about his role
    G. Inform Sam’s clinical supervisor about the incident
    H. Set some time aside to help Sam with his prioritisation skills
A

Answer: BDF
Rationale: This question deals with a colleague in difficulty. It is appropriate that you guide Sam to speak to his clinical supervisor to discuss these issues (B). It is important that patient care does not suffer and so it is appropriate that you suggest to Sam an alternative approach to ensuring completion of the urgent tasks (D). It would be useful to discuss with Sam what he is finding difficult as you may be able to provide some solutions for improving things as you both do the same job (F). You should not talk about Sam behind his back to his peers (A) or ask them to pick up his work to cover for him (E). It is better that Sam goes to his clinical supervisor rather than you go to them without speaking to Sam first (G) and equally it is their role to help Sam with his prioritisation skills and not yours (H). Option (C) is not appropriate if you both have the same role but only Sam is struggling.